ORTHOPEDIC MCQS ONLINE QUESTION BANK H3C

ORTHOPEDIC MCQS ONLINE QUESTION BANK H3C

 

 

 

Slide 1 Slide 2 Slide 3 Slide 4

 

 

 

Slide 5

A 45-year-old man has pain with overhead activity and discomfort when he lies on his shoulder at night. Plain radiographs of his humerus are shown in Slide 1. T1- and T2-weighted coronal images are shown in Slide 2 and Slide 3, respectively. A low- and high-power biopsy specimen is shown in Slide 4 and Slide 5, respectively. The best form of treatment would be:

 

1) Internal fixation and external beam irradiation

3) Preoperative chemotherapy and wide resection

2) External beam irradiation

5) Wide resection alone

4) Observation

 

The plain radiographs of the humerus show an intramedullary lesion that is mineralized. Notice that the cortices are intact. The T1-weighted coronal magnetic resonance image (MRI) shows a well-defined, low- signal lesion and the T2-weighted image shows a well-defined, high-signal lesion. Both sequences show a lobular (small circle) pattern to the lesion. The plain radiograph and MRI scan are consistent with a cartilage lesion. The lack of cortical changes indicates a diagnosis of enchondroma.

 

The biopsy specimen on low power shows lobules of cartilage surrounded by enchondral bone formation. The cartilage is hypocellular, and the nuclei are consistently small and round.

The diagnosis of enchondroma can be made with plain radiographs alone or plain radiographs and MRI scans. Treatment is observation with new radiographs in 3 to 6 months.Correct Answer: Observation

 

 

 

 

 

Slide 1 Slide 2 Slide 3 Slide 4

 

 

 

Slide 5

A 30-year-old woman has had a painless mass in the back of her knee for 6 months. She has a restricted range of motion. The plain radiographs are shown in Slide 1 and Slide 2. The biopsy specimen is shown in Slide 3, Slide 4, and Slide 5. The best treatment for this patient would be:

 

1) Observation

3) Wide resection and reconstruction

2) Preoperative chemotherapy followed by wide resection/reconstruction

5) Internal fixation followed by external beam irradiation

4) Preoperative chemotherapy and radiation followed by resection/reconstruction

 

Notice the heavily mineralized lesion on the surface of the bone. This is consistent with a parosteal osteosarcoma. There is a broad attachment to the bone and the tumor encircles the bone. The histologic section shows mature bone and a low-grade fibrous stroma (Grade 1 fibrosarcoma).

 

The diagnosis is parosteal osteosarcoma. Parosteal osteosarcomas are best treated by surgical resection without chemotherapy or radiation therapy.Correct Answer: Wide resection and reconstruction

 

 

 

 

 

Slide 1 Slide 2 Slide 3

A 58-year-old woman has diffuse pelvic and right hip pain. She has pain at rest and when she ambulates. The plain radiograph and computed tomography scan are shown in Slide 1 and Slide 2, respectively. A biopsy specimen is shown in Slide 3. The most likely diagnosis is:

 

1) Lymphoma

3) Multiple myeloma

2) Metastatic bone disease

5) Malignant fibrous histiocytoma

4) Chondrosarcoma

 

The plain radiograph shows a sclerotic lesion in the intertrochanteric region, ilium, ischium, and sacrum. The lesions are densely sclerotic. The biopsy specimen shows epithelial cells in clusters forming glands. These cells have completely replaced the bone marrow.

 

 

 

 

 

 

 

 

This patient has metastatic breast carcinoma.Correct Answer: Metastatic bone disease 3740. (2422) Q9-2883:

 

Slide 1 Slide 2 Slide 3

A 58-year-old woman has diffuse pelvic and right hip pain. She has pain at rest and when she ambulates. The plain radiograph and computed tomography scan are shown in Slide 1 and Slide 2, respectively. A biopsy specimen is shown in Slide 3. The most effective treatment would be:

 

1) Dynamic hip screw fixation of the femur and external beam irradiation

3) External beam irradiation

2) Intramedullary rod fixation of the femur and external beam irradiation

5) Resection of the proximal femur and custom replacement

4) Chemotherapy and external beam irradiation to painful sites

 

The plain radiograph shows a sclerotic lesion in the intertrochanteric region, ilium, ischium, and sacrum. The lesions are densely sclerotic. The biopsy specimen shows epithelial cells in clusters forming glands. These cells have completely replaced the bone marrow.

 

This patient has metastatic breast carcinoma.

 

The lesions in the proximal femur are completely sclerotic with no evidence of cortical bone destruction. The proximal femur is at low risk for fracture; therefore, surgery is not necessary. The best treatment is systemic chemotherapy and radiation to painful sites. Radiation alone is not adequate as the patient should be on systemic treatment (chemotherapy, hormonal therapy, or monoclonal antibody therapy).Correct Answer: Chemotherapy and external beam irradiation to painful sites

 

 

Slide 1 Slide 2

A 60-year-old woman has had severe pain in her leg for the past 8 months. The plain radiograph is shown in Slide 1 , and a biopsy of the tibia in Slide 2. The most likely diagnosis is:

 

1) Metastatic bone disease

3) Chondrosarcoma

2) Malignant fibrous histiocytoma

5) Lymphoma

4) Myeloma

 

The plain radiograph shows a lesion with multiple areas of bone lysis and areas of sclerosis. There is also thickening of the anterior and posterior cortices. The biopsy specimen shows large and small blue cells. Notice that the cells do not match the description of metastatic bone disease, myeloma, malignant fibrous histiocytoma, or chondrosarcoma. This patient has a lymphoma.

 

 

 

Metastatic bone disease - epithelial cells forming glands in a fibrous background Multiple myeloma - sheets of plasma cells

 

 

Malignant fibrous histiocytoma - storiform pattern of spindle cells and histiocytes Chondrosarcoma - cartilage forming tumor with atypical chondrocytes

Lymphoma

 

 

Uniform cells with large round nucleus and clumped chromatin Scant cytoplasm

Correct Answer: Lymphoma

 

 

Slide 1 Slide 2

A 60-year-old woman has had severe pain in her leg for the past 8 months. The plain radiograph is shown in Slide 1, and a biopsy of the tibia in Slide 2. The most appropriate treatment is:

 

1) Curettage and bone grafting

3) External beam irradiation and chemotherapy

2) Resection and prosthetic reconstruction

5) Above the knee amputation

4) Diphosphonate therapy

 

The plain radiograph shows a lesion with multiple areas of bone lysis and areas of sclerosis. There is also thickening of the anterior and posterior cortices. The biopsy specimen shows large and small blue cells. Notice that the cells do not match the description of metastatic bone disease, myeloma, malignant fibrous histiocytoma, or chondrosarcoma. This patient has a lymphoma.

 

 

 

Metastatic bone disease - epithelial cells forming glands in a fibrous background Multiple myeloma - sheets of plasma cells

 

 

Malignant fibrous histiocytoma - storiform pattern of spindle cells and histiocytes Chondrosarcoma - cartilage forming tumor with atypical chondrocytes

Lymphoma

 

 

Uniform cells with large round nucleus and clumped chromatin Scant cytoplasm

Correct Answer: External beam irradiation and chemotherapy

 

 

 

 

 

Slide 1 Slide 2 Slide 3 Slide 4

A 55-year-old man presented with a 12-month history of increasing low back pain. He experiences pain during the night and often has difficulty sitting. He has also noticed that in the last 9 months constipation has become an increasing problem. The plain radiographs and computerized tomography (CT) scan are shown (Slide 1 and Slide 2). Low- and high-power histologic sections are also presented (Slide 3 and Slide 4). The most appropriate treatment is:

 

1) Chemotherapy and radiation

3) Observation and protected weight bearing

2) External radiation

5) Wide resection

4) Chemotherapy and wide resection

 

The plain radiograph of the pelvis shows the sacrum and sacroiliac joints. The CT scan shows destruction of the sacrum with an anterior soft tissue mass. The low-power photomicrograph shows a lobular tumor. The high-power photomicrograph shows physaliferous cells in a myxoid background.

 

Both the CT scan and biopsy are consistent with a chordoma.

 

Chordoma is a low-grade malignant tumor that arises from notochordal cells. As a low-grade tumor, metastases occur late in the disease. Local failure is common following surgical resection. Progressive local disease accounts for the poor 10-year disease-free survival of only 25%.Correct Answer: Wide resection

 

 

3744. (2426) Q9-2887:

 

 

 

Slide 1 Slide 2 Slide 3 Slide 4

A 30-year-old attorney has a 6-month history of increasing knee pain. His pain occurs both with activity and at rest. He is otherwise healthy. Laboratory studies show a normal complete blood count, sedimentation rate, and chemistry values (including normal calcium and phosphorus levels). Anteroposterior and lateral radiographs are shown (Slide 1 and Slide 2). The low- and high-power hematoxylin and eosin sections are shown (Slide 3 and Slide 4). The most appropriate treatment is:

 

1) Observation

3) External beam irradiation

2) Curettage

5) Wide resection and reconstruction

4) Chemotherapy and external beam irradiation

 

The radiograph shows purely lytic bone destruction that fills the lateral metaphysis and extends to the articular surface. The lateral cortex is very thin and almost completely destroyed. The histologic sections show collection of giant cells and mononuclear cells.

Notice that the giant cells are uniformly scattered throughout the histologic section on the low power, in contrast to chondroblastoma where the giant cells are scattered throughout the mononuclear cells and that the nuclei of the mononuclear cells are the same as the nuclei of the giant cells.

 

The diagnosis is giant cell tumor of bone. The treatment of giant cell tumor of bone is complete curettage of the tumor. Wide exteriorization (extensive removal of the cortex over the lesion), curettage with hand and power instruments, and chemical cauterization is necessary to reduce the chance of local recurrence. Cement augmentation is often used to fill the large cavity following curettage.Correct Answer: Curettage

 

 

 

 

 

Slide 1 Slide 2 Slide 3

A 25-year-old man has a 4-month history of severe thigh and knee pain. He has pain both at rest and at night. He has tenderness on palpation of the thigh. He has been febrile and has an elevated white blood cell count. The anteroposterior and lateral radiographs of the femur are shown (Slide1). The high- and lower-power histologic sections are also shown (Slide 2) and (Slide 3). The most appropriate treatment is:

 

1) Observation

3) Methyl prednisolone acetate injection

2) Debridement and intravenous antibiotics

5) Chemotherapy and external beam irradiation

4) Wide resection

 

The plain radiographs show permeative lytic lesion in the femur with periosteal reaction. The diagnostic considerations based upon the age of the patient and radiographic findings would be:

Benign Malignant

Osteomyelitis Ewing's tumor Eosinophilic granuloma Osteosarcoma

Leukemia/lymphoma

 

The biopsy specimen shows a monotonous field of uniform blue cells. The biopsy is most consistent with Ewing's tumor.

Ewing's tumor is a high-grade malignancy with a propensity to metastasize to the lungs and bone marrow. The traditional treatment is multi-agent chemotherapy and external beam irradiation. Resection of the bone is used in selected patients in some centers.

 

Remember that there is a consistent chromosomal translocation, 11-22, in patients with Ewing's tumor [t (11;22)].Correct Answer: Chemotherapy and external beam irradiation

 

 

 

 

 

Slide 1 Slide 2

A 55-year-old man presented with a 4-month history of severe knee pain. The plain anteroposterior radiograph of the knee is shown (Slide 1). He has pain both at rest and at night. A staging evaluation was performed to determine of this was a metastatic lesion. A chest radiograph, chest computed tomography (CT) scan, and an abdominal CT scan were all negative. Serum studies showed a normal complete blood cell count, differential, chemistry group, and serum electrophoresis. A biopsy was performed and is shown (Slide 2). The most appropriate treatment is:

 

1) Observation

3) External beam irradiation

2) Internal fixation with external beam irradiation

5) Wide resection and chemotherapy

4) Chemotherapy and external beam irradiation

 

The radiographics show a destructive lesion with a mottled appearance: a mixture of bone destruction and bone formation.

 

The differentiated diagnosis of this destructive lesion is based on the radiographic features and the patients age. Differentiated diagnosis includes: metastatic bone disease, myeloma, lymphoma, chondrosarcoma, and malignant fibrous histiocytoma.

 

The biopsy specimen shows the characteristic features of malignant fibrous histiocytoma: a storiform pattern, histiocytes, and bizarre giant cells. The diagnosis is malignant fibrous histiocytoma. This is a high grade tumor and wide resection is necessary.Correct Answer: Wide resection and chemotherapy

 

 

 

 

 

 

Slide 1 Slide 2 Slide 3

A 60-year-old man presents with a 6-month history of increasing knee and distal thigh pain. He is otherwise healthy and has no history of prior malignancies. He has discomfort at night and at rest. The plain radiograph of the lesion is shown (Slide 1), and low and high power histologic sections from an open biopsy are shown (Slide 2 and Slide 3). The most appropriate treatment is:

 

1) Observation

3) Curettage and grafting

2) Core decompression

5) Wide resection and reconstruction

4) Curettage, internal fixation, and external beam irradiation

 

Dedifferentiated chondrosarcomas are high-grade cartilage tumors that are bimorphic (two distinct histologic components). Alongside the very low-grade cartilage tumor are areas of high-grade spindle cell sarcoma. The cartilage component is very low grade (grade ½, grade 1, or grade 2), while the high-grade component is an anaplastic spindle cell sarcoma; either osteosarcoma, fibrosarcoma, or malignant fibrous histiocytoma. The radiographs also reflect the histologic bimorphic component; area typical for low-grade chondrosarcoma with superimposed aggressive area. The aggressive component is usually represented by an area of:

 

 

Lytic bone destruction

 

 

Cortical bone erosion or typically destruction Soft tissue mass

The treatment of dedifferentiated chondrosarcoma is wide resection by either local resection or amputation. The margins must include a cuff of normal tissue secondary to the high grade nature of the tumor. The prognosis is poor with pulmonary metastases in 60% to 80% of patients.

 

Correct Answer: Wide resection and reconstruction

 

 

3748. (2430) Q9-2891:

Which of the following serum tests is most useful in the diagnosis of Pagetâs disease:

 

1) Parathyroid hormone

3) Alkaline phosphatase

2) Calcium level

5) Lactic dehydrogenase

4) Phosphate level

 

Paget disease is a remodeling disease of bone. The osteoclast is the responsible cell. There is an increase in size, number, and activity. The treatment of Paget disease is aimed at the activity of the osteoclast. Diphosphonates are the mainstay of treatment.

 

The alkaline phosphatase level is elevated in 95% of patients. Collagen breakdown products can also be detected in the urine â hydroxy proline or N-telopeptide. Following treatment, there is a reduction in serum alkaline phosphate levels.Correct Answer: Alkaline phosphatase

 

 

 

 

 

Slide 1 Slide 2 Slide 3 Slide 4

 

 

 

Slide 5

A 30-year-old man presents with an 18-month history of pain in his leg. The pain is over the mid-section of the tibia and occurs both at rest and activity. There no history of trauma or infections. The plain radiographs are shown (Slide 1 and Slide2) and a biopsy of the lesion is shown (Slide 3, Slide 4 and Slide 5). The most appropriate treatment is:

 

1) Debridement and intravenous antibiotics

3) Curettage, cortical bone grafting, and internal fixation

2) Observation

5) Wide resection and reconstruction

4) Preoperative chemotherapy and wide resection

 

Adamantinoma most commonly occurs in the tibia and has multiple areas of bone destruction. There is often intervening bone sclerosis between the areas of bone destruction. The biopsy specimens shows the typical features of adamantinoma: epithelial cells in fibrous stroma.

 

The treatment of adamantinoma is wide resection. This lesion is a low grade malignancy with late metastases. There is no role for chemotherapy or radiation in patients with localized disease.Correct Answer: Wide resection and reconstruction

 

 

3750. (2542) Q9-3013:

Which of the following is the most common skeletal site of metastatic bone disease:

 

1) Proximal femur

3) Cervical spine

2) Pelvis

5) Ribs

4) Thoracolumbar spine

 

The most common sites of metastatic bone disease are the spine, pelvis, ribs, skull, and proximal limb girdles. The spine is the most common site and the thoracolumbar spine accounts for approximately 70% of spinal metastases.Correct Answer: Thoracolumbar spine

 

Which of the following is the correct histological description of metastatic bone disease:

 

1) Sheets of cells with an eccentric nucleus and a perinuclear clear zone

3) Clumps of round cells in a fibrous background

2) A mosaic pattern of broad irregular osteoid seams

5) Diffuse, noncleaved large cells permeating bone trabeculae

4) Storiform pattern of rounded and spindle cells

 

The characteristic histological appearance of metastatic bone disease is clumps of round cells in an organoid pattern in a fibrous stroma.

The other responses describe common conditions:

 

 

 

 

Myeloma: Sheets of cells with an eccentric nucleus and a perinuclear clear zone Lymphoma: Diffuse, noncleaved large cells permeating bone trabeculae Malignant fibrous histiocytoma: Storiform pattern of rounded and spindle cells Pagetâs disease: Mosaic pattern of broad irregular osteoid seams

 

Correct Answer: Clumps of round cells in a fibrous background

 

 

3752. (2544) Q9-3015:

Which of the following cells are responsible for the bone destruction in metastatic breast carcinoma:

 

1) Mast cells

3) Osteoclasts

2) Osteocytes

5) Histiocytes

4) Carcinoma cells

 

The bone resorption in both metastatic bone disease and multiple myeloma is caused by the osteoclasts. In both conditions, the tumor cells secrete a substance that recruits and activates the osteoclasts, which then resorb bone.Correct Answer: Osteoclasts

 

 

3753. (2545) Q9-3016:

A woman with newly diagnosed breast cancer has multiple liver and pulmonary metastases and a lytic lesion in the midshaft of the left femur with 20% to 25% cortical bone destruction in a symmetric manner involving the femoral endosteal surfaces. She has no pain with weight bearing. Which of the following is the most accepted method to treat patients with femoral metastasis:

 

1) External beam irradiation alone

3) Intramedullary rod fixation and external beam irradiation

2) External beam irradiation and diphosphonate therapy

5) Curettage and plate fixation, external beam irradiation

4) Intramedullary rod fixation, diphosphonate therapy, and external beam irradiation

 

Patients with metastases to long bones may require prophylactic fixation. Plain radiographs are evaluated for the amount of cortical bone destruction and patients are asked if they have weight bearing pain. When cortical bone destruction exceeds 50% of the cortical diameter, the risk of fracture is high. When prominent weight bearing pain is present, the risk of fracture is also greater. This patient has a small amount of cortical bone destruction and no weight bearing pain. She can be managed nonoperatively.

 

External beam irradiation is used to halt the bone destruction by killing the tumor cells. Diphosphonate therapy is used to stop the osteoclastic bone resorption both in the femoral shaft and at other sites.

Correct Answer: External beam irradiation and diphosphonate therapy

 

Which of the following three variables concerning soft tissue sarcomas would portend the worst prognosis:

 

1) Low grade, superficial, and size less than 5 cm

3) High grade, superficial, and size less than 5 cm

2) Low grade, deep, and size less than 5 cm

5) Low grade, deep, and greater than 5 cm

4) High grade, deep, and greater than 5 cm

 

 

The variables that adversely effect prognosis in soft tissue sarcomas are: High grade

 

 

Depth below the fascia Size greater than 5 cm

High-grade soft tissue sarcomas that are superficial (above the fascia) have a better prognosis than high-grade lesions below the fascia.

Correct Answer: High grade, deep, and greater than 5 cm

 

 

3755. (2547) Q9-3018:

Which of the following soft tissue lesions has the greater risk of local recurrence following excision:

 

1) Lipoma

3) Nodular fasciitis

2) Schwannoma

5) Angiolipoma

4) Extra-abdominal desmoid

 

Extra-abdominal desmoid tumors are prone to local recurrence with a rate as high as 50% in many series. External beam irradiation and low-dose chemotherapy have been used to halt progressive growth. The addition of external beam irradiation in selected cases improves the local control rate to approximately 75%.

 

Lipomas, schwannomas, nodular fasciitis, and angiolipoma have a low risk of local recurrence. Correct Answer: Extra-abdominal desmoid

 

3756. (2548) Q9-3019:

Which of the following soft tissue tumors has the least risk of pulmonary metastases:

 

1) Epithelioid sarcoma

3) Pleomorphic liposarcoma

2) Synovial sarcoma

5) Clear cell sarcoma

4) Myxoid liposarcoma

 

Epithelioid sarcoma, synovial sarcoma, pleomorphic liposarcoma, and clear cell sarcoma are high-grade soft tissue sarcomas. The risk of pulmonary metastases in these soft tissue sarcomas is higher than 50%.

 

Myxoid liposarcoma is an intermediate grade liposarcoma with a metastatic rate of between 20% to 30%. Correct Answer: Myxoid liposarcoma

Which of the following genetic abnormalities may occur in polyostotic fibrous dysplasia:

 

1) Chromosomal translocation, 11-22

3) EXT1, EXT2 genes

2) Activating mutation, GNAS 1 gene

5) t(12;16) translocation

4) t(X;18)(p11.2;q11.2)

 

In polyostotic fibrous dysplasia, an activating mutation of the GNAS 1 gene has been found. There may also be over expression of the C-fos proto-oncogene.

The other responses refer to specific conditions:

 

 

Ewingâs tumor: chromosomal translocation, 11-22 Multiple exostoses: EXT1, EXT2 genes

 

 

Synovial sarcoma: t(X;18)(p11.2;q11.2) Myxoid liposarcoma: t(12;16) translocation

Correct Answer: Activating mutation, GNAS 1 gene

 

 

3758. (2550) Q9-3022:

Which of the following describes the histological features of fibrous dysplasia:

 

1) Woven bone in a fibrous stroma with prominent osteoblastic rimming

3) Fascicles of atypical fibroblasts in a herringbone pattern

2) Spicules of woven bone in a cellular fibrous stroma

5) Storiform pattern of spindle cells with scattered small giant cells

4) Monotonous bland spindle cell proliferation with abundant collagen

 

The histological features of fibrous dysplasia are:

 

 

Spicules of woven bone in cellular fibrous stroma: the woven bone may form patterns of Chinese letters or alphabet soup The woven bone lacks osteoblastic rimming

 

Nodules of cartilage may be present The other responses include:

 

 

 

 

Osteofibrous dysplasia: Woven bone in a fibrous stroma with prominent osteoblastic rimming Desmoplastic fibroma: Monotonous bland spindle cell proliferation with abundant collagen Nonossifying fibroma: Storiform pattern of spindle cells with scattered small giant cells Fibrosarcoma: Fascicles of atypical fibroblasts in a herringbone pattern

 

Correct Answer: Spicules of woven bone in a cellular fibrous stroma

 

Which of the following describes the radiographic features of nonossifying fibroma of bone:

 

1) Metaphyseal, cortical bone destruction, ivory-like bone formation

3) Metaphyseal, eccentric, thinned cortex, sclerotic rim

2) Metaphyseal-epiphyseal, lytic, poorly marginalized

5) Diaphyseal, multiple lucent defects with intervening sclerosis

4) Epiphyseal, geographic lytic area, sclerotic rim

 

 

The characteristic radiographic appearance of nonossifying fibroma is: Metaphyseal

 

 

 

Cortically based Lucent area Sclerotic rim

 

Overlying cortex is thinned but intact

The other responses are for common bone lesions:

 

 

 

 

Osteosarcoma: metaphyseal, cortical bone destruction, ivory like bone formation Giant cell tumor: metaphyseal-epiphyseal, lytic, poorly marginalized Chondroblastoma: epiphyseal, geographic lytic area, sclerotic rim Adamantinoma: multiple lucent defects with intervening sclerosis

 

 

 

 

Correct Answer: Metaphyseal, eccentric, thinned cortex, sclerotic rim 3760. (2578) Q9-3051:

 

Slide 1

A 15-year-old boy has a soft, painful mass over the proximal portion of his elbow. The lateral plain radiograph is shown (Slide). The most likely diagnosis:

 

1) Heterotopic ossification

3) Hemangioma

2) Synovial sarcoma

5) Tumoral calcinosis

4) Calcific tendonitis

 

The lateral radiograph shows multiple perfectly round radiodensities scattered throughout the soft tissues. These radiodensities are phleboliths, and they are virtually diagnostic of a soft tissue hemangioma.

Surgically, hemangiomas are difficult to treat because they have a high incidence of local recurrence. Symptomatic treatment with anti-inflammatory medications and compression dressing is the first-line therapy. An interventional radiologist uses sclerosing therapy to treat patients who still have significant symptoms following symptomatic treatment. A sclerosing agent such as alcohol is used to close the numerous blood vessels.

 

Correct Answer: Hemangioma

 

 

 

 

 

Slide 1

A 15-year-old boy has a soft, painful mass over the proximal portion of his elbow. The lateral plain radiograph is shown (Slide). Which of the following is the best form of treatment:

 

1) Angiography and sclerosing therapy

3) Chemotherapy

2) External beam irradiation

5) Chemotherapy and surgical resection

4) Surgical resection

 

The lateral radiograph shows multiple perfectly round radiodensities scattered throughout the soft tissues. These radiodensities are phleboliths, and they are virtually diagnostic of a soft tissue hemangioma.

Surgically, hemangiomas are difficult to treat because they have a high incidence of local recurrence. Symptomatic treatment with anti-inflammatory medications and compression dressing can be used. An interventional radiologist uses sclerosing therapy to treat patients who still have significant symptoms following symptomatic treatment. A sclerosing agent such as alcohol is used to close the numerous blood vessels.

 

 

 

 

 

 

Correct Answer: Angiography and sclerosing therapy 3762. (2580) Q9-3053:

 

Slide 1 Slide 2

A 35-year-old man has pain and limited pronation-supination of his forearm. His plain radiograph (Slide 1) and computed tomography scan (Slide 2) are shown. The most likely diagnosis is:

 

1) Sclerotic synovial sarcoma

3) Heterotopic ossification

2) Parosteal osteosarcoma

5) Parosteal osteoma

4) Parosteal lipoma

 

The plain radiograph shows heavy bone formation lateral to the proximal radius. This bone formation could be caused by a parosteal osteosarcoma, heterotopic ossification, or a parosteal osteoma.

The computed tomography scan shows the mature bone formation that is more mineralized at the periphery than in the center. The bone spans the radius and the ulna. The zonal pattern of greater mineralization at the periphery compared to the center is characteristic of heterotopic ossification.

 

Correct Answer: Heterotopic ossification

 

 

 

 

 

Slide 1 Slide 2

A 35-year-old man has pain and limited pronation-supination of his forearm. His plain radiograph (Slide 1) and computed tomography scan (Slide 2) are shown. Appropriate treatment for this patient might include:

 

1) Preoperative chemotherapy and wide resection

3) External beam irradiation alone

2) Removal of the bridging bone and low-dose external beam irradiation

5) Above elbow amputation

4) Chemotherapy and radiation

 

The plain radiograph shows heavy bone formation lateral to the proximal radius. This bone formation could be caused by a parosteal osteosarcoma, heterotopic ossification, or a parosteal osteoma.

The computed tomography scan shows the mature bone formation that is more mineralized at the periphery than in the center. The bone spans the radius and the ulna. The zonal pattern of greater mineralization at the periphery compared to the center is characteristic of heterotopic ossification.

 

Heterotopic ossification treatment is either observation or removal to improve motion and function. Recurrence of the heterotopic bone may occur, and patients are often treated with low-dose external beam irradiation (usually one fraction of 800 cGy).

 

 

 

Correct Answer: Removal of the bridging bone and low-dose external beam irradiation 3764. (2582) Q9-3055:

 

Slide 1

A 10-year-old boy has shoulder pain following a fall onto his shoulder. The plain radiograph is shown (Slide). Which of the following is the most likely diagnosis:

 

1) Osteosarcoma

3) Aneurysmal bone cyst

2) Ewingâs tumor

5) Fibrous dysplasia

4) Unicameral bone cyst

 

The plain radiograph shows a lytic lesion in the proximal humeral metaphysis. The medial and lateral cortices are thinned but intact (except for the minimally displaced fracture).

This radiograph is virtually diagnostic of unicameral bone cyst. The treatment is aspiration of the cyst and injection with methylprednisolone acetate.

Correct Answer: Unicameral bone cyst

 

 

 

 

 

Slide 1

A 10-year-old boy has shoulder pain following a fall onto his shoulder. The plain radiograph is shown (Slide). Which of the following is the most accepted treatment:

 

1) Preoperative chemotherapy followed by wide resection

3) Wide resection and reconstruction

2) External beam irradiation

5) Aspiration and cortisone injection

4) Internal fixation and bone grafting

 

The plain radiograph shows a lytic lesion in the proximal humeral metaphysis. The medial and lateral cortices are thinned but intact (except for the minimally displaced fracture).

This radiograph is virtually diagnostic of unicameral bone cyst. The treatment is aspiration of the cyst and injection with methylprednisolone acetate.

 

 

 

Correct Answer: Aspiration and cortisone injection 3766. (2584) Q9-3057:

 

Slide 1

A 10-year-old boy has shoulder pain following a fall onto his shoulder. The plain radiograph is shown (Slide). Which of the following statements is true concerning the diagnosis:

 

1) Unicameral bone cysts are low-grade malignancies.

3) Unicameral bone cysts are aggressive benign neoplasms.

2) Unicameral bone cysts are high-grade malignancies.

5) Unicameral bone cysts are aggressive, benign non-neoplastic processes.

4) Unicameral bone cysts are self-limited, benign non-neoplastic processes.

 

The plain radiograph shows a lytic lesion in the proximal humeral metaphysis. The medial and lateral cortices are thinned but intact (except for the minimally displaced fracture).

This radiograph is virtually diagnostic of unicameral bone cyst. The treatment is aspiration of the cyst and injection with methylprednisolone acetate.

 

Unicameral bone cysts are self-limited, benign non-neoplastic processes. They are not neoplasms. Correct Answer: Unicameral bone cysts are self-limited, benign non-neoplastic processes.

 

 

 

 

Slide 1

A 28-year-old woman has a hard mass over the extensor compartment of her forearm. The mass measures 6 cm and is located in the subcutaneous tissue. The mass is low signal on T1-weighted sequences and heterogeneous on T2-weighted sequences with low, moderate, and high areas. A needle biopsy specimen is shown (Slide). The most likely diagnosis is:

 

1) Fibrosarcoma

3) Synovial sarcoma

2) Malignant fibrous histiocytoma

5) Atypical lipoma

4) Extra-abdominal desmoid tumor

 

The imaging studies show a mass in the patient. The signal characteristics are nonspecific and do not aid the clinician in deciding whether the mass is benign or malignant.

The biopsy specimen shows a uniform population of fibroblasts and dense collagen deposition, which are characteristic of an extra-abdominal desmoid tumor.

Correct Answer: Extra-abdominal desmoid tumor

 

 

 

 

 

Slide 1

A 50-year-old man has a 5-cm popliteal space mass that is low signal on T1-weighted sequences and high signal on T2-weighted sequences. The biopsy specimen is shown (Slide). The most likely diagnosis is:

 

1) Malignant fibrous histiocytoma

3) Schwannoma

2) Synovial sarcoma

5) Liposarcoma

4) Desmoid tumor

 

A large soft tissue mass that is located below the fascia should raise the suspicion of a sarcoma. All sarcomas have a common imaging pattern - low signal on T1-weighted sequences and high signal on T2-weighted sequences.

The biopsy specimen shows the features of a schwannoma with Antoni A and B areas. Note the cellular area and the loose myxoid area.

 

Antoni A - Compact spindle cells Twisted nuclei

 

Indistinct cytoplasmic borders

 

Short bundles or interlacing fascicles

 

Antoni B - Haphazard spindle cells in loose matrix, large vessels that may be filled with thrombus Correct Answer: Schwannoma

 

3769. (2587) Q9-3060:

 

 

 

Slide 1

A 30-year-old woman has a 10-cm arm mass in located in the subcutaneous tissue. The mass is homogeneously high signal on T1-weighted images and moderate signal on T2-weighted images. A needle biopsy is shown (Slide). The most likely diagnosis is:

 

1) Liposarcoma

3) Epithelioid sarcoma

2) Malignant fibrous histiocytoma

5) Desmoid tumor

4) Lipoma

 

The mass is located in the subcutaneous tissue and could represent a benign process such as a lipoma or a sarcoma. Lesions that are high signal on T1-weighted images and moderate signal on T2-weighted sequences have the characteristics of fat. The biopsy shows mature fat cells with some intervening areas of fibrous tissue. This patient has a lipoma.Correct Answer: Lipoma

 

Which of the following soft tissue sarcomas is most prone to metastasizing to lymph nodes:

 

1) Myxoid liposarcoma

3) Clear cell sarcoma

2) Malignant fibrous histiocytoma

5) Pleomorphic liposarcoma

4) Fibrosarcoma

 

 

Several soft tissue sarcomas are prone to spread to lymph nodes: Synovial sarcoma

 

 

 

Epithelioid sarcoma Rhabdomyosarcoma Clear cell sarcoma

 

Correct Answer: Clear cell sarcoma

 

 

3771. (2589) Q9-3062:

Synovial sarcomas may display genetic abnormalities. Which of the following is the most common genetic alteration in synovial sarcoma:

 

1) Giant marker and ring chromosomes

3) Translocation between chromosomes 11 and 22

2) Translocation between chromosomes 12 and 16

5) Translocation between chromosomes 9 and 22

4) Translocation between chromosomes X and 18

 

Sarcoma

Well-differentiated liposarcoma Myxoid liposarcoma

Ewing's sarcoma/

primitive neuroectodermal tumor Synovial sarcoma

Myxoid chondrosarcoma

Genetic Alteration

Giant marker and ring chromosomes Translocation between chromosomes 12 and 16

 

Translocation between chromosomes 11 and 22 Translocation between chromosomes X and 18 Translocation between chromosomes 9 and 22

Correct Answer: Translocation between chromosomes X and 18

 

 

 

 

 

Slide 1 Slide 2

A 55-year-old man has a 10-cm thigh mass that is low signal on T1-weighted images and high signal on T2-weighted images. A needle biopsy is performed and shown in Slide 1 and Slide 2. The most likely diagnosis is:

 

1) Soft tissue metastasis

3) Schwannoma

2) Myositis ossificans (heterotopic ossification)

5) Collagenous fibroma

4) Malignant fibrous histiocytoma

 

A large soft tissue mass that is located below the fascia should raise the suspicion of a sarcoma. All sarcomas have a common imaging pattern - low signal on T1-weighted sequences and high signal on T2-weighted sequences.

The biopsy shows the typical storiform pattern of malignant fibrous histiocytoma. Notice the spindle cells with different sizes and shapes of the nuclei (pleomorphism). There are also cells with round and vesicular nuclei.

 

 

 

Correct Answer: Malignant fibrous histiocytoma 3773. (2591) Q9-3064:

 

Slide 1

A 45-year-old man has a 5-cm hard mass over the extensor compartment of his forearm located below the fascia. A needle biopsy is shown (Slide). The most likely diagnosis is:

 

1) Fibrosarcoma

3) Schwannoma

2) Nodular fasciitis

5) Soft tissue Ewingâs tumor

4) Extra-abdominal desmoid tumor

 

A soft tissue mass that is located below the fascia should raise the suspicion of a sarcoma. All sarcomas have a common imaging pattern - low signal on T1-weighted sequences and high signal on T2-weighted sequences.

The biopsy shows spindle cells (fibroblasts) arranged in fascicles. Some of the nuclei are cut longitudinally, causing a spindle-shaped nuclei, whereas other nuclei are cut in cross section and appear round. The interlacing of the fascicles is termed a herringbone pattern. This patient has a fibrosarcoma.

 

Correct Answer: Fibrosarcoma

 

 

 

 

 

Slide 1

A 60-year-old woman has a 15-cm soft tissue mass on her posterior thigh. The mass is low signal on T1-weighted images and high signal on T2-weighted images and is located below the fascia. A needle biopsy is shown (Slide). The most likely diagnosis is:

 

1) Lipoma

3) Liposarcoma

2) Atypical lipoma

5) Desmoid tumor

4) Schwannoma

 

A soft tissue mass that is located below the fascia should raise the suspicion of a sarcoma. All sarcomas have a common imaging pattern - low signal on T1-weighted sequences and high signal on T2-weighted sequences.

The needle biopsy shows a cellular lesion that has marked pleomorphism of the cells (different sizes and shapes of the nuclei). Some cells have intracytoplasmic fat globules; these cells are lipoblasts. The presence of the lipoblasts and the pleomorphic spindles cells is compatible with a liposarcoma. For examination purposes, one would not be able to distinguish this mass from a malignant fibrous histiocytoma.

 

 

 

 

Correct Answer: Liposarcoma 3775. (2593) Q9-3066:

 

Slide 1

A 30-year-old man has a 5-cm lesion in the deep tissues of his foot. The lesion is located below the fascia and is low signal on T1-weighted images and high signal on T2-weighted images. A needle biopsy is shown (Slide). The most likely diagnosis is:

 

1) Lipoma

3) Nodular fasciitis

2) Heterotopic ossification

5) Schwannoma

4) Synovial sarcoma

 

A soft tissue mass that is located below the fascia should raise the suspicion of a sarcoma. All sarcomas have a common imaging pattern - low signal on T1-weighted sequences and high signal on T2-weighted sequences.

The needle biopsy shows a lesion that has spindle cells and cells with round nuclei or epithelial-appearing cells. The epithelial cells either are arranged in clusters or might appear to be forming glands. This bimorphic pattern of spindle cells and epithelial cells is consistent with a synovial sarcoma.

 

Correct Answer: Synovial sarcoma

 

 

 

 

 

Slide 1 Slide 2

A 23-year-old man has a 3-cm firm mass on the ulnar border of his hand. The mass is low signal on T1-weighted images and high signal on T2-weighted images. A biopsy is shown in Slide 1 and Slide 2. The most likely diagnosis is:

 

1) Nodular fasciitis

3) Foreign body granuloma

2) Schwannoma

5) Sebaceous cyst

4) Epithelioid sarcoma

 

A rare malignant tumor known as epithelioid sarcoma may occur in the upper extremity in young patients.

The biopsy shows a nodule on low power with an area of central necrosis. On high power, one sees rounded cells with eosinophilic cytoplasm. This is an epithelioid sarcoma.

 

 

 

Correct Answer: Epithelioid sarcoma 3777. (2595) Q9-3068:

 

Slide 1

A 10-year-old child has a 10-cm thigh mass that has grown rapidly over a 1-month period. The mass is deep to the fascia and is low signal on T1-weighted images and high signal on T2-weighted images. A biopsy is shown (Slide). The most likely diagnosis is:

 

1) Nodular fasciitis

3) Rhabdomyosarcoma

2) Heterotopic ossification

5) Atypical lipoma

4) Schwannoma

 

A rapidly growing mass in a child that is deep to the fascia is most likely a malignant tumor. Common sarcomas in young children include rhabdomyosarcoma, soft tissue Ewing's tumor, and primitive neuroectodermal tumor. All of these sarcomas look the same on magnetic resonance imaging - low signal on T1-weighted images and high signal on T2-weighted images.

 

These sarcomas also have a similar appearance on hematoxylin and eosin staining. There are clusters on small, round, blue cells. This patient has a rhabdomyosarcoma - other acceptable answers include soft tissue Ewing's tumor and primitive neuroectodermal tumor.

 

Correct Answer: Rhabdomyosarcoma

 

 

Slide 1 Slide 2 Slide 3 Slide 4

A 65-year-old woman has had severe arm and shoulder pain for 4 months. Her plain radiographs are shown in Slide 1 and Slide

2. Sections from a needle biopsy are shown in Slide 3 and Slide 4. The most likely diagnosis is:

 

1) Metastatic bone disease

3) Lymphoma

2) Multiple myeloma

5) Chondrosarcoma

4) Malignant fibrous histiocytoma

 

The plain radiographs show a destructive lesion with a permeative pattern of bone destruction (multiple small lytic areas) and areas of bone formation - a mixed pattern of bone destruction and bone formation. The differential diagnosis based upon the radiographs alone in this age group (ages 40 to 80 years) includes:

 

 

 

 

Metastatic bone disease Multiple myeloma Lymphoma

 

Primary mesenchymal bone tumors:

   Chondrosarcoma

   Malignant fibrous histiocytoma

The biopsy specimen shows epithelial cells clumped together in an organoid pattern. This patient has metastatic breast carcinoma.

Correct Answer: Metastatic bone disease

 

 

Slide 1 Slide 2 Slide 3 Slide 4

A 65-year-old woman has had severe arm and shoulder pain for 4 months. Her plain radiographs are shown in Slide 1 and Slide

2. Sections from a needle biopsy are shown in Slide 3 and Slide 4. Which of the following is the optimal treatment method:

 

1) Wide resection and prosthetic reconstruction followed by irradiation

3) Wide resection and prosthetic reconstruction

2) Internal fixation, external beam irradiation, and diphosphonate therapy

5) Observation

4) External beam irradiation and diphosphonate therapy

 

The plain radiographs show a destructive lesion with a permeative pattern of bone destruction (multiple small lytic areas) and areas of bone formation - a mixed pattern of bone destruction and bone formation. The differential diagnosis based upon the radiographs alone in this age group (ages 40 to 80 years) includes:

 

 

 

 

Metastatic bone disease Multiple myeloma Lymphoma

 

Primary mesenchymal bone tumors:

Chondrosarcoma

Malignant fibrous histiocytoma

The biopsy specimen shows epithelial cells clumped together in an organoid pattern. This patient has metastatic breast carcinoma.

There is between 25% and 50% cortical bone destruction. Note that the medial and anterior cortices are completely intact. This patient can be treated with external beam irradiation and diphosphonate therapy with a low risk of fracture (about 10%). Although internal fixation is not necessary, if the patient does not want to risk having a fracture, then fixation can be performed with postoperative external beam irradiation and diphosphonate therapy.

 

Correct Answer: External beam irradiation and diphosphonate therapy

 

 

Slide 1 Slide 2 Slide 3 Slide 4

A 65-year-old woman has had severe arm and shoulder pain for 4 months. Her plain radiographs are shown in Slide 1 and Slide

2. Sections from a needle biopsy are shown in Slide 3 and Slide 4. Which of the following is the correct disease process:

 

1) Metabolic bone disease

3) Benign neoplasm

2) Infection

5) Metastatic bone disease

4) Primary malignant mesenchymal tumor

 

The plain radiographs show a destructive lesion with a permeative pattern of bone destruction (multiple small lytic areas) and areas of bone formation - a mixed pattern of bone destruction and bone formation. The differential diagnosis based upon the radiographs alone in this age group (ages 40 to 80 years) includes:

 

 

 

 

Metastatic bone disease Multiple myeloma Lymphoma

 

Primary mesenchymal bone tumors:

   Chondrosarcoma

   Malignant fibrous histiocytoma

The biopsy specimen shows epithelial cells clumped together in an organoid pattern. This patient has metastatic breast carcinoma.

Metastases can be confused with metabolic disorders such as Paget's disease or primary mesenchymal tumors such as chondrosarcoma.

Correct Answer: Metastatic bone disease

 

 

Slide 1 Slide 2 Slide 3 Slide 4

A 65-year-old woman has had severe arm and shoulder pain for 4 months. Her plain radiographs are shown in Slide 1 and Slide

2. Sections from a needle biopsy are shown in Slide 3 and Slide 4. The final common denominator for the bone destruction in this patient is:

 

1) Parathyroid-related hormone (PTHrP)

3) Osteoprotegerin (OPG)

2) Receptor activator of nuclear activator ligand (RANKL)

5) Insulin-like growth factor 1 (IGF1)

4) Transforming growth factor beta (TGFB)

 

The plain radiographs show a destructive lesion with a permeative pattern of bone destruction (multiple small lytic areas) and areas of bone formation - a mixed pattern of bone destruction and bone formation. The differential diagnosis based upon the radiographs alone in this age group (ages 40 to 80 years) includes:

 

 

 

 

Metastatic bone disease Multiple myeloma Lymphoma

 

Primary mesenchymal bone tumors:

   Chondrosarcoma

   Malignant fibrous histiocytoma

The biopsy specimen shows epithelial cells clumped together in an organoid pattern. This patient has metastatic breast carcinoma.

The breast carcinoma cells release PTHrP, which stimulates the osteoblasts to release RANKL, which activates the osteoclast progenitor cells. Remember that RANKL is the final common denominator for osteoclast activation. These cells then become osteoclasts and resorb bone. With the bone resorption, there is release of TGFB and IGF1 that stimulate the tumors cells to release more PTHrP, causing more bone destruction.

 

Remember that RANKL binds to the RANK receptor on the osteoclast progenitor cell and that OPG is a competitive decoy inhibitor to RANKL.

Correct Answer: Receptor activator of nuclear activator ligand (RANKL)

 

 

Slide 1 Slide 2 Slide 3 Slide 4

A 65-year-old woman has had severe arm and shoulder pain for 4 months. Her plain radiographs are shown in Slide 1 and Slide

2. Sections from a needle biopsy are shown in Slide 3 and Slide 4. Which of the following is expressed by tumor cells:

 

1) Parathyroid-related hormone (PTHrP)

3) Osteoprotegerin (OPG)

2) Receptor activator of nuclear activator ligand (RANKL)

5) Insulin-like growth factor 1 (IGF1)

4) Transforming growth factor beta (TGFB)

 

The plain radiographs show a destructive lesion with a permeative pattern of bone destruction (multiple small lytic areas) and areas of bone formation - a mixed pattern of bone destruction and bone formation. The differential diagnosis based upon the radiographs alone in this age group (ages 40 to 80 years) includes:

 

 

 

 

Metastatic bone disease Multiple myeloma Lymphoma

 

Primary mesenchymal bone tumors:

Chondrosarcoma

Malignant fibrous histiocytoma

The biopsy specimen shows epithelial cells clumped together in an organoid pattern. This patient has metastatic breast carcinoma.

The breast carcinoma cells release PTHrP, which stimulates the osteoblasts to release RANKL, which activates the osteoclast progenitor cells. Remember that RANKL is the final common denominator for osteoclast activation. These cells then become osteoclasts and resorb bone. With the bone resorption, there is release of TGFB and IGF1 that stimulate the tumors cells to release more PTHrP, causing more bone destruction.

 

Remember that RANKL binds to the RANK receptor on the osteoclast progenitor cell and that OPG is a competitive decoy inhibitor to RANKL.

Correct Answer: Parathyroid-related hormone (PTHrP)

 

 

Slide 1 Slide 2 Slide 3 Slide 4

A 65-year-old woman has had severe arm and shoulder pain for 4 months. Her plain radiographs are shown in Slide 1 and Slide

2. Sections from a needle biopsy are shown in Slide 3 and Slide 4. Which of the following is secreted by the osteoblasts after receiving a molecular signal from the tumor cells:

 

1) Parathyroid-related hormone (PTHrP)

3) Osteoprotegerin (OPG)

2) Receptor activator of nuclear activator ligand (RANKL)

5) Insulin-like growth factor 1 (IGF1)

4) Transforming growth factor beta (TGFB)

 

The plain radiographs show a destructive lesion with a permeative pattern of bone destruction (multiple small lytic areas) and areas of bone formation - a mixed pattern of bone destruction and bone formation. The differential diagnosis based upon the radiographs alone in this age group (ages 40 to 80 years) includes:

 

 

 

 

Metastatic bone disease Multiple myeloma Lymphoma

 

Primary mesenchymal bone tumors:

   Chondrosarcoma

   Malignant fibrous histiocytoma

The biopsy specimen shows epithelial cells clumped together in an organoid pattern. This patient has metastatic breast carcinoma.

The breast carcinoma cells release PTHrP, which stimulates the osteoblasts to release RANKL, which activates the osteoclast progenitor cells. Remember that RANKL is the final common denominator for osteoclast activation. These cells then become osteoclasts and resorb bone. With the bone resorption, there is release of TGFB and IGF1 that stimulate the tumors cells to release more PTHrP, causing more bone destruction.

 

Remember that RANKL binds to the RANK receptor on the osteoclast progenitor cell and that OPG is a competitive decoy inhibitor to RANKL.

Correct Answer: Receptor activator of nuclear activator ligand (RANKL)

 

 

Slide 1 Slide 2 Slide 3 Slide 4

A 65-year-old woman has had severe arm and shoulder pain for 4 months. Her plain radiographs are shown in Slide 1 and Slide

2. Sections from a needle biopsy are shown in Slide 3 and Slide 4. Which of the following molecules is capable of decreasing osteoclastic activation:

 

1) Parathyroid-related hormone (PTHrP)

3) Osteoprotegerin (OPG)

2) Receptor activator of nuclear activator ligand (RANKL)

5) Insulin-like growth factor 1 (IGF1)

4) Transforming growth factor beta (TGFB)

 

The plain radiographs show a destructive lesion with a permeative pattern of bone destruction (multiple small lytic areas) and areas of bone formation - a mixed pattern of bone destruction and bone formation. The differential diagnosis based upon the radiographs alone in this age group (ages 40 to 80 years) includes:

 

 

 

 

Metastatic bone disease Multiple myeloma Lymphoma

 

Primary mesenchymal bone tumors:

Chondrosarcoma

Malignant fibrous histiocytoma

The biopsy specimen shows epithelial cells clumped together in an organoid pattern. This patient has metastatic breast carcinoma.

The breast carcinoma cells release PTHrP, which stimulates the osteoblasts to release RANKL, which activates the osteoclast progenitor cells. Remember that RANKL is the final common denominator for osteoclast activation. These cells then become osteoclasts and resorb bone. With the bone resorption, there is release of TGFB and IGF1 that stimulate the tumors cells to release more PTHrP, causing more bone destruction.

 

Remember that RANKL binds to the RANK receptor on the osteoclast progenitor cell and that OPG is a competitive decoy inhibitor to RANKL.

Correct Answer: Osteoprotegerin (OPG)

 

 

Slide 1 Slide 2 Slide 3 Slide 4

A 65-year-old woman has had severe arm and shoulder pain for 4 months. Her plain radiographs are shown in Slide 1 and Slide

2. Sections from a needle biopsy are shown in Slide 3 and Slide 4.Which of the following is released during bone resorption:

 

1) Parathyroid-related hormone (PTHrP)

3) Osteoprotegerin (OPG)

2) Receptor activator of nuclear activator ligand (RANKL)

5) Smad protein

4) Transforming growth factor beta (TGFB)

 

The plain radiographs show a destructive lesion with a permeative pattern of bone destruction (multiple small lytic areas) and areas of bone formation - a mixed pattern of bone destruction and bone formation. The differential diagnosis based upon the radiographs alone in this age group (ages 40 to 80 years) includes:

 

 

 

 

Metastatic bone disease Multiple myeloma Lymphoma

 

Primary mesenchymal bone tumors:

Chondrosarcoma

Malignant fibrous histiocytoma

The biopsy specimen shows epithelial cells clumped together in an organoid pattern. This patient has metastatic breast carcinoma.

The breast carcinoma cells release PTHrP, which stimulates the osteoblasts to release RANKL, which activates the osteoclast progenitor cells. Remember that RANKL is the final common denominator for osteoclast activation. These cells then become osteoclasts and resorb bone. With the bone resorption, there is release of TGFB and IGF1 that stimulate the tumors cells to release more PTHrP, causing more bone destruction.

 

Remember that RANKL binds to the RANK receptor on the osteoclast progenitor cell and that OPG is a competitive decoy inhibitor to RANKL.

Correct Answer: Transforming growth factor beta (TGFB)

 

 

Slide 1 Slide 2

A 14-year-old girl twisted her knee while playing lacrosse. Her anteroposterior (Slide 1) and lateral (Slide 2) radiographs are shown. The most likely cause of this condition is:

 

1) Benign bone neoplasm

3) Benign bone developmental condition

2) Malignant bone neoplasm

5) Stress fracture

4) Metabolic bone disease

 

The plain radiographs show an oval lucent lesion in the metaphysis of the proximal tibia. Note that the lesion is based on the metaphysic. This is the classic appearance of a small nonossifying fibroma. In this case, there is no sclerotic rim.

Nonossifying fibromas of bone are best considered as variants of normal growth. Nonossifying fibromas are rare in children younger than 2 years old and commonly occur at the sites of muscle attachment.

 

 

 

 

 

Correct Answer: Benign bone developmental condition 3787. (2754) Q9-3252:

 

Slide 1 Slide 2

A 14-year-old girl twisted her knee while playing lacrosse. Her anteroposterior (Slide 1) and lateral (Slide 2) radiographs are shown. The most likely diagnosis is:

 

1) Osteomyelitis

3) Bone island

2) Stress fracture

5) Enchondroma

4) Nonossifying fibroma

 

The plain radiographs show an oval lucent lesion in the metaphysis of the proximal tibia. Note that the lesion is based on the metaphysic. This is the classic appearance of a small nonossifying fibroma. In this case, there is no sclerotic rim.

Nonossifying fibromas of bone are best considered as variants of normal growth. Nonossifying fibromas are rare in children younger than 2 years old and commonly occur at the sites of muscle attachment.

Correct Answer: Nonossifying fibroma

 

 

Slide 1 Slide 2

A 14-year-old girl twisted her knee while playing lacrosse. Her anteroposterior (Slide 1) and lateral (Slide 2) radiographs are shown. The most appropriate treatment is:

 

1) Curettage and bone grafting

3) En bloc resection and grafting

2) Aspiration and cortisone injection

5) Observation

4) Computed tomography-guided radiofrequency ablation

 

The plain radiographs show an oval lucent lesion in the metaphysis of the proximal tibia. Note that the lesion is based on the metaphysic. This is the classic appearance of a small nonossifying fibroma. In this case, there is no sclerotic rim.

Nonossifying fibromas of bone are best considered as variants of normal growth. Nonossifying fibromas are rare in children younger than 2 years old and commonly occur at the sites of muscle attachment.

Most nonossifying fibromas are asymptomatic and can be treated with observation. In this case, the nonossifying fibroma is so small that observation with a new radiograph in 3 months is all that is necessary. No activity restrictions are needed.

 

 

 

Correct Answer: Observation 3789. (2756) Q9-3254:

 

Slide 1

A 14-year-old boy developed pain following a collision while playing soccer. His radiograph is shown (Slide). The most likely cause of this condition is:

 

1) Benign bone neoplasm

3) Benign bone developmental condition

2) Malignant bone neoplasm

5) Stress fracture

4) Metabolic bone disease

 

The plain radiograph shows an oval lucent lesion in the metaphysis of the proximal tibia with a sclerotic rim. Note that the lesion is based on the metaphysic. This is the classic appearance of a small nonossifying fibroma. There is also a stress fracture at the superior lateral border (sclerotic band).

 

Nonossifying fibromas of bone are best considered as variants of normal growth. Nonossifying fibromas are rare in children younger than 2 years old and commonly occur at the sites of muscle attachment.

Correct Answer: Benign bone developmental condition

 

 

 

 

 

Slide 1

A 14-year-old boy developed pain following a collision while plating soccer. His radiograph is shown (Slide). The most likely cause of this condition is:

 

1) Osteomyelitis

3) Bone island

2) Stress fracture

5) Enchondroma

4) Nonossifying fibroma

 

The plain radiograph shows an oval lucent lesion in the metaphysis of the proximal tibia with a sclerotic rim. Note that the lesion is based on the metaphysic. This is the classic appearance of a small nonossifying fibroma. There is also a stress fracture at the superior lateral border (sclerotic band).

 

Nonossifying fibromas of bone are best considered as variants of normal growth. Nonossifying fibromas are rare in children younger than 2 years old and commonly occur at the sites of muscle attachment.

Correct Answer: Nonossifying fibroma

 

 

Slide 1 Slide 2

A 55-year-old man has a large soft tissue mass in his arm. The T1- and T2-weighted images are shown in Slide 1 and Slide 2. The most likely diagnosis is:

 

1) Benign soft tissue neoplasm

3) Muscle hernia

2) Malignant soft tissue neoplasm

5) Arteriovenous malformation

4) Synovial cyst

 

The T1-weighted image shows a large high-signal mass in the patient's arm. The signal quality exactly matches the subcutaneous fat. Note on the T2-weighted image with fat suppression that the mass completely suppresses (or turns black or low signal).

This patient has a lipoma or a benign soft tissue neoplasm. The patient can be treated with either observation or removal of the lipoma. The lipoma can be removed with a marginal or intralesional margin.

The other lesions refer to common soft tissue processes:

 

 

A malignant soft tissue neoplasm is low signal on T1-weighted images and high signal on T2-weighted images. A muscle hernia exactly follows muscle signal.

 

A synovial cyst is uniformly dark signal on T1-weighted images and high signal on T2-weighted images.

 

An arteriovenous malformation shows serpiginous channels that are low signal on T1-weighted images and high signal on T2-weighted images.

Correct Answer: Benign soft tissue neoplasm

 

 

Slide 1 Slide 2

A 55-year-old man has a large soft tissue mass in his arm. The T1- and T2-weighted images are shown in Slide 1 and Slide 2. The most likely diagnosis is:

 

1) Intramuscular lipoma

3) Malignant fibrous histiocytoma

2) Glenohumeral ganglion

5) Hematoma

4) Hemangioma

 

The T1-weighted image shows a large high-signal mass in the patient's arm. The signal quality exactly matches the subcutaneous fat. Note on the T2-weighted image with fat suppression that the mass completely suppresses (or turns black or low signal).

This patient has a lipoma or a benign soft tissue neoplasm. The patient can be treated with either observation or removal of the lipoma. The lipoma can be removed with a marginal or intralesional margin.

The other lesions refer to common soft tissue processes:

 

A malignant fibrous histiocytoma is low signal on T1-weighted images and high signal on T2-weighted images.

 

A hematoma shows fluid signal and is heterogeneous, with a low signal on T1-weighted images and high signal on T2-weighted images.

 

A glenohumeral ganglion is uniformly dark signal on T1-weighted images and high signal on T2-weighted images.

 

An arteriovenous malformation shows serpiginous channels that are low on T1-weighted imaged and high signal on T2-weighted images.

Correct Answer: Intramuscular lipoma

 

 

 

 

Slide 1 Slide 2 Slide 3

A 55-year-old man has a large soft tissue mass in his arm. The T1- and T2-weighted images are shown in Slide 1 and Slide 2. A biopsy is shown in Slide 3. Reasonable treatment options include:

 

1) Preoperative radiation followed by wide resection

3) Shoulder disarticulation

2) Wide resection followed by external beam irradiation

5) Preoperative chemotherapy and radiation followed by wide resection

4) Excision with a marginal margin

 

The T1-weighted image shows a large high-signal mass in the patientâs arm. The signal quality exactly matches the subcutaneous fat. Note on the T2-weighted image with fat suppression that the mass completely suppresses (or turns black or low signal).

This patient has a lipoma or a benign soft tissue neoplasm. The patient can be treated with either observation or removal of the lipoma. The lipoma can be removed with a marginal or intralesional margin.

 

 

 

 

 

Correct Answer: Excision with a marginal margin 3794. (2761) Q9-3259:

 

Slide 1 Slide 2

A 55-year-old man has a large soft tissue mass in his arm. The T1- and T2-weighted images are shown in Slide 1 and Slide 2. The abnormal signal in the deltoid muscle is most likely:

 

1) Simple fluid

3) Malignant tumor cells

2) Complex fluid

5) Fibrous tissue

4) Fat

 

The T1-weighted image shows a large high-signal mass in the patient's arm. The signal quality exactly matches the subcutaneous fat. Note on the T2-weighted image with fat suppression that the mass completely suppresses (or turns black or low signal).

This process is benign fat cells.

This patient has a lipoma or a benign soft tissue neoplasm. The patient can be treated with either observation or removal of the lipoma. The lipoma can be removed with a marginal or intralesional margin.

Correct Answer: Fat

 

 

Slide 1 Slide 2

A 45-year-old woman has a large soft tissue mass in her deltoid muscle, which is located in the shoulder region. The T1- and T2-weighted images are shown in Slide 1 and Slide 2. The most likely diagnosis is:

 

1) Benign soft tissue neoplasm

3) Muscle hernia

2) Malignant soft tissue neoplasm

5) Arteriovenous malformation

4) Synovial cyst

 

The T1-weighted image shows a large high-signal mass in the deltoid muscle. The signal quality exactly matches the subcutaneous fat. Note on the T2-weighted image with fat suppression that the mass completely suppresses (or turns black or low signal).

This patient has a lipoma or a benign soft tissue neoplasm. The patient can be treated with either observation or removal of the lipoma. The lipoma can be removed with a marginal or intralesional margin.

The other lesions refer to common soft tissue processes:

 

 

A malignant soft tissue neoplasm is low signal on T1-weighted images and high signal on T2-weighted images. A muscle hernia exactly follows muscle signal.

 

A synovial cyst is uniformly dark signal on T1-weighted images and high signal on T2-weighted images.

 

An arteriovenous malformation shows serpiginous channels that are low signal on T1-weighted images and high signal on T2-weighted images.

Correct Answer: Benign soft tissue neoplasm

 

 

Slide 1 Slide 2

A 45-year-old woman has a large soft tissue mass in her deltoid muscle, which is located in the shoulder region. The T1- and T2-weighted images are shown in Slide 1 and Slide 2. The most likely diagnosis is:

 

1) Intramuscular lipoma

3) Malignant fibrous histiocytoma

2) Glenohumeral ganglion

5) Hematoma

4) Hemangioma

 

The T1-weighted image shows a large high-signal mass in the deltoid muscle. The signal quality exactly matches the subcutaneous fat. Note on the T2-weighted image with fat suppression that the mass completely suppresses (or turns black or low signal).

This patient has a lipoma or a benign soft tissue neoplasm. The patient can be treated with either observation or removal of the lipoma. The lipoma can be removed with a marginal or intralesional margin.

The other lesions refer to common soft tissue processes:

 

A malignant fibrous histiocytoma is low signal on T1-weighted images and high signal on T2-weighted images.

 

A hematoma shows fluid signal and is heterogeneous, with a low signal on T1-weighted images and high signal on T2-weighted images.

 

A glenohumeral ganglion is uniformly dark signal on T1-weighted images and high signal on T2-weighted images.

 

An arteriovenous malformation shows serpiginous channels that are low on T1-weighted imaged and high signal on T2-weighted images.

Correct Answer: Intramuscular lipoma

 

 

 

 

Slide 1 Slide 2 Slide 3

A 45-year-old woman has a large soft tissue mass in her deltoid muscle, which is located in the shoulder region. The T1- and T2-weighted images are shown in Slide 1 and Slide 2. A biopsy is shown in Slide 3. Reasonable treatment options include:

 

1) Preoperative radiation followed by wide resection

3) Shoulder disarticulation

2) Wide resection followed by external beam irradiation

5) Preoperative chemotherapy and radiation followed by wide resection

4) Excision with a marginal margin

 

The T1-weighted image shows a large high-signal mass in the deltoid muscle. The signal quality exactly matches the subcutaneous fat. Note on the T2-weighted image with fat suppression that the mass completely suppresses (or turns black or low signal).

This patient has a lipoma or a benign soft tissue neoplasm. The patient can be treated with either observation or removal of the lipoma. The lipoma can be removed with a marginal or intralesional margin.

 

 

 

 

 

Correct Answer: Excision with a marginal margin 3798. (2765) Q9-3263:

 

Slide 1 Slide 2

A 45-year-old woman has a large soft tissue mass in her deltoid muscle, which is located in the shoulder region. The T1- and T2-weighted images are shown in Slide 1 and Slide 2. The abnormal signal in the deltoid muscle is most likely:

 

1) Simple fluid

3) Malignant tumor cells

2) Complex fluid

5) Fibrous tissue

4) Fat

 

The T1-weighted image shows a large high-signal mass in the deltoid muscle. The signal quality exactly matches the subcutaneous fat. Note on the T2-weighted image with fat suppression that the mass completely suppresses (or turns black or low signal).

This process is benign fat cells.

This patient has a lipoma or a benign soft tissue neoplasm. The patient can be treated with either observation or removal of the lipoma. The lipoma can be removed with a marginal or intralesional margin.

Correct Answer: Fat

 

The classic triad of Hand-Schüller-Christian disease is:

 

1) Pelvis lesion, enophthalmos, diabetes insipidus

3) Skull lesion, enophthalmos, acromegaly

2) Pelvis lesion, exophthalmos, acromegaly

5) Scapula lesion, diabetes insipidus, acromegaly

4) Skull lesion, exophthalmos, diabetes insipidus

 

Hand-Schüller-Christian disease has a constellation of findings; the classic triad is a skull lesion, exophthalmos, and diabetes insipidus.

Hand-Schüller-Christian disease used to be called histiocytosis X. It is now called Langerhans cell histiocytosis. Patients may have solitary bone involvement, multiple bone involvement, or bone and soft tissue involvement. The liver, spleen, and lymph nodes may be involved.

 

Correct Answer: Skull lesion, exophthalmos, diabetes insipidus

 

 

3800. (2776) Q9-3274:

Which of the following is true concerning Langerhans cell histiocytosis (eosinophilic granuloma) of the spine:

 

1) Posterior element involvement is more common than anterior involvement.

3) Asymmetric vertebral collapse has a higher risk of long-term deformity.

2) Cervical spine involvement is rare.

5) Short-term bracing is effective treatment.

4) Neurological deficits are common.

 

Patients with Langerhans cell histiocytosis may develop spinal involvement.

One should remember these points from a recent study by Garg and colleagues.

 

Cervical spine involvement is common.

 

Anterior elements are involved in 95% of patients.

 

 

The most common symptom in patients is dull low back pain. Neurological defects are uncommon.

 

 

Symmetric vs. asymmetric collapse is not a prognostic factor for long-term deformity. Short-term bracing is effective.

 

Approximately 10% of patients may need spinal deformity correction.

Correct Answer: Short-term bracing is effective treatment.

 

 

3801. (2777) Q9-3275:

Which of the following are classic findings in Langerhans cell histiocytosis of the spine:

 

1) Vertebral collapse, preservation of the disk space, absence of a soft tissue mass

3) Vertebral collapse, preservation of the disk space, anterior soft tissue mass

2) Vertebral collapse, disk space narrowing, absence of a soft tissue mass

5) Vertebral expansion, disk space narrowing, presence of a soft tissue mass

4) Vertebral expansion, disk space narrowing, absence of a soft tissue mass

 

Patients with Langerhans cell histiocytosis may develop spinal involvement.

One should remember these points from a recent study by Garg and colleagues.

 

Cervical spine involvement is common.

 

 

Multilevel disease is common especially in the cervical and lumbar regions. Anterior elements are involved in 95% of patients.

 

 

Vertebral body collapse is common and may be symmetric or asymmetric. The most common symptom in patients is dull low back pain.

 

Neurological defects are uncommon.

 

 

Symmetric vs. asymmetric collapse is not a prognostic factor for long-term deformity. Short-term bracing is effective.

 

Approximately 10% of patients may need spinal deformity correction.

Correct Answer: Vertebral collapse, preservation of the disk space, absence of a soft tissue mass

 

 

3802. (2778) Q9-3276:

Which of the following is true concerning Langerhans cell histiocytosis (eosinophilic granuloma) of the spine:

 

1) Dull back pain without neurological symptoms is the most common symptom.

3) Narrowing of disk space is common.

2) The most common site of involvement is the posterior element.

5) Multicentric spine involvement is rare.

4) Vertebral body reconstruction with correction of deformity is necessary with more than 50% collapse.

 

Patients with Langerhans cell histiocytosis may develop spinal involvement.

One should remember these points from a recent study by Garg and colleagues.

 

Cervical spine involvement is common.

 

 

Multilevel disease is common especially in the cervical and lumbar regions. Anterior elements are involved in 95% of patients.

 

 

Vertebral body collapse is common and may be symmetric or asymmetric. The most common symptom in patients is dull low back pain.

 

Neurological defects are uncommon.

 

 

Symmetric vs. asymmetric collapse is not a prognostic factor for long-term deformity. Short-term bracing is effective.

 

Approximately 10% of patients may need spinal deformity correction.

Correct Answer: Dull back pain without neurological symptoms is the most common symptom.

 

 

Slide 5

A 35-year-old man has recurrent knee joint effusions, pain, and limited range of motion. A plain radiograph is shown in Slide 1, a T1-weighted coronal and sagittal sequence magnetic resonance image (MRI) in Slide 2 and Slide 3, and an axial T2-weighted image in Slide 4. A biopsy of the synovium is shown in Slide 5. The most likely diagnosis is:

 

1) Tuberculosis

3) Pigmented villonodular synovitis

2) Coccidiomycosis

5) Rheumatoid arthritis

4) Synovial chondromatosis

 

The plain radiograph shows erosion at the margin of the joint. The MRI scans show proliferation of the synovium.

The biopsy shows a mononuclear cell infiltrate. The cells are round to oval with pink cytoplasm. Hemosiderin-laden macrophages are also present.

This patient has pigmented villonodular synovitis. The treatment is synovectomy, which can be performed either arthroscopically or open. Patients with recurrent lesions or extensive disease are also offered external beam irradiation.

Correct Answer: Pigmented villonodular synovitis

 

 

Slide 5

A 35-year-old man has recurrent knee joint effusions, pain, and limited range of motion. A plain radiograph is shown in Slide 1, a T1-weighted coronal and sagittal sequence magnetic resonance image (MRI) in Slide 2 and Slide 3, and an axial T2-weighted image in Slide 4. A biopsy of the synovium is shown in Slide 5. The most appropriate treatment for this patient is:

 

1) Debridement and antibiotics

3) Observation

2) Rheumatic medications

5) Triple antibiotic therapy alone

4) Synovectomy

 

The plain radiograph shows erosion at the margin of the joint. The MRI scans show proliferation of the synovium.

The biopsy shows a mononuclear cell infiltrate. The cells are round to oval with pink cytoplasm. Hemosiderin-laden macrophages are also present.

This patient has pigmented villonodular synovitis. The treatment is synovectomy, which can be performed either arthroscopically or open. Patients with recurrent lesions or extensive disease are also offered external beam irradiation.

Correct Answer: Synovectomy

 

 

3805. (2785) Q9-3283:

Which of the following is the most common location of synovial chondromatosis:

 

1) Hip

3) Shoulder

2) Knee

5) Ankle

4) Elbow

 

Synovial chondromatosis results when metaplasia of the subsynovial fibroblasts produces nodules of cartilage in the synovial membrane.

 

The most common location is the knee. The process may also occur in the hip, shoulder, and elbow. Correct Answer: Knee

 

 

 

 

Slide 1 Slide 2 Slide 3

A 22-year-old man has intermittent aching pain in his left hip. His plain radiograph is shown in Slide 1 and the T1- and T2-weighted coronal magnetic resonance image (MRI) scans are shown in Slide 2 and Slide 3. The most likely diagnosis is:

 

1) Pigmented villonodular synovitis

3) Synovial chondromatosis

2) Rheumatoid arthritis

5) Heterotopic ossification

4) Tumoral calcinosis

 

Synovial chondromatosis results when metaplasia of the subsynovial fibroblasts produces nodules of cartilage in the synovial membrane.

The most common location is the knee. The process may also occur in the hip, shoulder, and elbow.

Numerous small and large round, mineralized loose bodies are present in the plain radiograph. The MRI scan shows a joint effusion (high signal on T2-weighted images) and low signal masses. The T1-weighted images also show the numerous small low-signal masses.

 

This patient has synovial chondromatosis. Correct Answer: Synovial chondromatosis

 

3807. (2787) Q9-3285:

 

 

 

Slide 1 Slide 2 Slide 3

A 22-year-old man has intermittent aching pain in his left hip. His plain radiograph is shown in Slide 1 and the T1- and T2-weighted coronal magnetic resonance image (MRI) scans are shown in Slide 2 and Slide 3. Which of the following is the most beneficial treatment:

 

1) Protected weight bearing with crutches

3) Synovectomy

2) Removal of a parathyroid adenoma

5) Chemotherapy

4) External beam irradiation

 

This patient has synovial chondromatosis. Treatment for patients with this condition includes synovectomy with removal of as many of the loose bodies as possible.Correct Answer: Synovectomy

 

 

Slide 1 Slide 2

A 12-year-old girl has occasional thigh pain. Her plain radiographs are shown in Slide 1 and a biopsy specimen in Slide 2. The most likely diagnosis is:

 

1) Aneursymal bone cyst

3) Osteosarcoma

2) Osteoblastoma

5) Osteofibrous dysplasia

4) Fibrous dysplasia

 

The plain radiographs show a lesion in the medullary cavity. Symmetric expansion of the bone is present. The cortices have been remodeled, but they are intact. The lesion has a mineralization pattern that is consistent with the ground glass appearance of fibrous dysplasia.

 

The biopsy specimen shows fragments of woven bone in a highly cellular fibrous stroma.

This child has fibrous dysplasia. One should remember that the genetic abnormality in fibrous dysplasia is caused by a mutation that affects the alpha subunit of the cell membrane-bound G protein (Gs alpha mutation).

Correct Answer: Fibrous dysplasia

 

 

Slide 1 Slide 2

A 12-year-old girl has occasional thigh pain. Her plain radiographs are shown in Slide 1 and a biopsy specimen in Slide 2. Which of the following is the most likely genetic defect:

 

1) FGF receptor 3

3) Gs alpha mutation

2) EXT1, EXT2 genes

5) Translocation between chromosomes X and 18

4) COMP, type IX collagen, (COL9A2)

 

The plain radiographs show a lesion in the medullary cavity. Symmetric expansion of the bone is present. The cortices have been remodeled, but they are intact. The lesion has a mineralization pattern that is consistent with the ground glass appearance of fibrous dysplasia.

 

The biopsy specimen shows fragments of woven bone in a highly cellular fibrous stroma.

This child has fibrous dysplasia. One should remember that the genetic abnormality in fibrous dysplasia is caused by a mutation that affects the alpha subunit of the cell membrane-bound G protein (Gs alpha mutation).

The other answers refer to specific entities:

 

 

FGF receptor 3 - achondroplasia EXT1, EXT2 - multiple exostoses

 

 

COMP, type IX collagen, (COL9A2) - multiple epiphyseal dysplasia Translocation between chromosomes X and 18 - synovial sarcoma

 

 

 

 

 

 

 

Correct Answer: Gs alpha mutation 3810. (2949) Q9-3452:

 

Slide 1 Slide 2 Slide 3

A 10-year-old child has a mass over the anterior aspect of the tibia. A plain radiograph is shown in Slide 1 and a biopsy specimen in Slide 2 and Slide 3. The most likely diagnosis is:

 

1) Osteomyelitis

3) Osteofibrous dysplasia

2) Lymphoma

5) Adamantinoma

4) Fibrous dysplasia

 

The lateral radiograph of the tibia shows a large anterior cortex lucency. Notice that there is expansion of the anterior cortex and a gentle bow to the tibia, which is diagnostic for osteofibrous dysplasia. The biopsy specimen shows spicules of woven bone that are lined by osteoblasts. This is also diagnostic of osteofibrous dysplasia. Patients are generally treated with observation.Correct Answer: Osteofibrous dysplasia

 

 

 

 

 

Slide 1 Slide 2 Slide 3

A 10-year-old child has a mass over the anterior aspect of the tibia. A plain radiograph is shown in Slide 1 and a biopsy specimen in Slide 2 and Slide 3. Which of the following would be the most appropriate treatment:

 

1) Debridement and antibiotic therapy

3) Chemotherapy and external beam irradiation

2) Preoperative chemotherapy followed by resection/reconstruction

5) Observation

4) External beam irradiation alone

 

The lateral radiograph of the tibia shows a large anterior cortex lucency. Notice that there is expansion of the anterior cortex and a gentle bow to the tibia, which is diagnostic for osteofibrous dysplasia. The biopsy specimen shows spicules of woven bone that are lined by osteoblasts. This is also diagnostic of osteofibrous dysplasia. Patients are generally treated with observation.Correct Answer: Observation

 

 

3812. (2977) Q9-3483:

Malignant fibrous histiocytoma of bone:

 

1) Can be confused for fibrosarcoma of bone

3) Are usually low grade

2) Usually produces a significant osseous response

5) Rarely metastasizes

4) Are most commonly seen in the first 2 decades of life similar to Ewingâs sarcoma

 

Malignant fibrous histiocytoma of bone is a lytic tumor of bone, which is often high grade. The peak age of patients is 50 years of age, and metastasis occurs in approximately 50% of patients.Correct Answer: Can be confused for fibrosarcoma of bone

 

 

3813. (2978) Q9-3484:

Chondromyxoid fibroma:

 

1) Frequently dedifferentiates into a malignant tumor if untreated

3) May cross the physis to involve the epiphysis

2) Rarely involves the physis

5) Often presents by fracture

4) Has a microscopic picture in which the chondral element predominates

 

Chondromyxoid fibroma has once been reported to dedifferentiate to chondrosarcoma. The tumor commonly abuts the physis, and is microscopically predominated by the myxoid element. Chondromyxoid fibroma rarely presents with a fracture.Correct Answer: May cross the physis to involve the epiphysis

 

Which of the following is the most important prognostic factor in a patient with a soft tissue sarcoma:

 

1) Location (superficial or deep)

3) Grade

2) Size

5) Gender

4) Age

 

The most important prognostic factor in patients with soft tissue sarcomas is the stage of the tumor. Staging takes into account the presence of metastases, grade, size, and depth.

 

The following factors are listed in descending order of impact on survival: Presence of metastases

 

 

 

Grade Size Depth

 

The risk of pulmonary metastases is much higher in patients with high-grade tumors compared to low-grade tumors. Patients with soft tissue sarcomas that are located above the fascia have a better prognosis than those located below the fascia.

Correct Answer: Grade

 

 

3815. (2996) Q9-3502:

Which of the following is the most important prognostic factor in a patient with a soft tissue sarcoma:

 

1) Grade

3) Depth

2) Size

5) Gender

4) Presence of metastases

 

The most important prognostic factor in patients with soft tissue sarcomas is the stage of the tumor. Staging takes into account the presence of metastases, grade, size, and depth.

 

The following factors are listed in descending order of impact on survival: Presence of metastases

 

 

 

Grade Size Depth

 

The risk of pulmonary metastases is much higher in patients with high-grade tumors compared to low-grade tumors. Patients with soft tissue sarcomas that are located above the fascia have a better prognosis than those located below the fascia.

Correct Answer: Presence of metastases

 

 

 

1) Depth

3) Size

2) Gender

5) Encapsulation

4) Age

 

The most important prognostic factor in patients with soft tissue sarcomas is the stage of the tumor. Staging takes into account the presence of metastases, grade, size, and depth.

 

The following factors are listed in descending order of impact on survival: Presence of metastases

 

 

 

Grade Size Depth

 

The risk of pulmonary metastases is much higher in patients with high-grade tumors compared to low-grade tumors. Patients with soft tissue sarcomas that are located above the fascia have a better prognosis than those located below the fascia.

Correct Answer: Size

 

 

3817. (2998) Q9-3504:

Lymph node metastases are common in which of the following soft tissue tumors:

 

1) Extra-abdominal desmoid tumor

3) Malignant fibrous histiocytoma

2) Nodular fasciitis

5) Epitheloid sarcoma

4) Liposarcoma

 

 

Lymph node metastases may occur in the following soft tissue sarcomas: Lymph nodes (these lesions can have regional metastases):

 

 

 

 

Synovial sarcoma Epithelioid sarcoma Rhabdomyosarcoma Clear cell Sarcoma

 

 

Epitheloid sarcoma is prone to metastasize to regional lymph nodes and to the skin and subcutaneous tissue. Correct Answer: Epitheloid sarcoma

 

3818. (2999) Q9-3505:

Lymph node metastases are common in which of the following soft tissue sarcomas:

 

1) Malignant fibrous histiocytoma

3) Liposarcoma

2) Fibrosarcoma

5) Hemangiopericytoma

4) Synovial sarcoma

 

 

Lymph node metastases may occur in the following soft tissue sarcomas: Lymph nodes (these lesions can have regional metastases):

 

 

 

 

Synovial sarcoma Epithelioid sarcoma Rhabdomyosarcoma Clear cell Sarcoma

 

Epitheloid sarcoma is prone to metastasize to regional lymph nodes and to the skin and subcutaneous tissue. Correct Answer: Synovial sarcoma

 

 

3819. (3000) Q9-3506:

Soft tissue Ewing's sarcoma has which of the following genetic abnormalities:

 

1) Giant and marker ring chromosomes

3) Translocation between 11 and 22

2) Translocation between chromosomes 12 and 16

5) Translocation between 9 and 22

4) Translocation between X and 18

 

Several soft tissue sarcomas have well-recognized genetic abnormalities:

Well-differentiated liposarcoma giant marker and ring chromosomes Myxoid liposarcoma translocation between 12 and 16 Ewing's/primitive neuroectodermal tumors translocation between 11 and 22 Synovial sarcoma translocation between X and 18

Myxoid chondrosarcoma translocation between 9 and 22 Correct Answer: Translocation between 11 and 22

 

 

3820. (3001) Q9-3507:

Synovial sarcoma has which of the following chromosomal alterations:

 

1) Giant marker and ring chromosomes

3) Translocation between chromosomes 11 and 22

2) Translocation between chromosomes 12 and 16

5) Translocation between chromosomes 9 and 22

4) Translocation between chromosomes X and 18

 

Several soft tissue sarcomas have well-recognized genetic abnormalities:

Well-differentiated liposarcoma giant marker and ring chromosomes Myxoid liposarcoma translocation between 12 and 16 Ewing's/primitive neuroectodermal tumors translocation between 11 and 22 Synovial sarcoma translocation between X and 18

Myxoid chondrosarcoma translocation between 9 and 22 Correct Answer: Translocation between chromosomes X and 18

 

 

3821. (3002) Q9-3508:

Which of the following tumors commonly occurs in the hand:

 

1) Extra-abdominal desmoid tumor

3) Myxoid chondrosarcomas

2) Liposarcoma

5) Epitheloid sarcoma

4) Fibrosarcoma

 

Epitheloid sarcoma has a propensity to occur in the hand and upper extremity. This particular tumor is also prone to lymph node and skin/subcutaneous metastases.

Common locations include:

A. Location: Upper and lower extremities - 60%

  1. Thigh is most common location

  2. Specific tumors and site predilection

    1. Epitheloid sarcoma - most common soft tissue sarcoma of the upper extremity

       

    2. Synovial sarcoma -most common soft tissue sarcoma of the foot and ankle Correct Answer: Epitheloid sarcoma

3822. (3050) Q9-3557:

In determining prognosis of a patient with a bone sarcoma, which of the following is the most important variable:

 

1) Grade of the tumor

3) Stage

2) Location

5) Age of the patient

4) Size

 

 

The factors that affect prognosis are often reflected in staging systems: Presence of metastases

 

 

Grade of the tumor Size of the tumor

 

Remember that stage trumps all other factors. When stage is not an answer, the prognostic factors are: Presence of metastases

 

 

Grade of the tumor Size of the tumor

 

 

For bone tumors in the AJCC system, > 8 cm is worse Site of the tumor

 

Spine and pelvic tumors generally have a worse prognosis Important points to remember:

 

Osteosarcoma

 

Response to chemotherapy is one of the most important prognostic factors

 

 

99% necrosis is best, 90% to 99% is less favorable, <90% is a poor prognostic sign Retinoblastoma gene (Rb1) is an association

 

Some studies have shown a poorer prognosis with elevated serum alkaline phosphatase or lactate dehydrogenase (AP, LDH)- this is controversial

Correct Answer: Stage

 

 

3823. (3051) Q9-3558:

Which of the following is the most important prognostic factor in a patient with osteosarcoma:

 

1) Size

3) Presence of metastases

2) Location

5) Grade

4) Age

 

 

The factors that affect prognosis are often reflected in staging systems: Presence of metastases

 

 

Grade of the tumor Size of the tumor

 

Remember that stage trumps all other factors. When stage is not an answer, the prognostic factors are: Presence of metastases

 

 

Grade of the tumor Size of the tumor

 

 

For bone tumors in the AJCC system, > 8 cm is worse Site of the tumor

 

Spine and pelvic tumors generally have a worse prognosis Important points to remember:

 

Osteosarcoma

 

Response to chemotherapy is one of the most important prognostic factors

 

 

99% necrosis is best, 90% to 99% is less favorable, <90% is a poor prognostic sign Retinoblastoma gene (Rb1) is an association

 

Some studies have shown a poorer prognosis with elevated serum alkaline phosphatase or lactate dehydrogenase (AP, LDH) - this is controversial

Correct Answer: Presence of metastases

 

 

3824. (3052) Q9-3559:

A 13-year-old boy has a destructive lesion in the distal femur. A magnetic resonance imaging scan shows cortical destruction and a moderate-sized soft tissue mass (5.5 cm). Needle biopsy shows a high-grade sarcoma. Which of the following studies should be performed to determine the stage of the tumor:

 

1) Bone marrow biopsy, computed tomography (CT) chest, and technetium bone scan

3) CT chest and technetium bone scan

2) CT chest and abdomen and technetium bone scan

5) Skeletal survey and CT chest and abdomen

4) Skeletal survey, CT chest, and positron emission tomography scan

 

 

The factors that affect prognosis are often reflected in staging systems: Presence of metastases

 

 

Grade of the tumor Size of the tumor

 

Remember that stage trumps all other factors. When stage is not an answer, the prognostic factors are: Presence of metastases

 

 

Grade of the tumor Size of the tumor

 

 

For bone tumors in the AJCC system, > 8 cm is worse Site of the tumor

 

Spine and pelvic tumors generally have a worse prognosis Important points to remember:

 

Osteosarcoma

 

Response to chemotherapy is one of the most important prognostic factors

 

 

99% necrosis is best, 90% to 99% is less favorable, <90% is a poor prognostic sign Retinoblastoma gene (Rb1) is an association

 

Some studies have shown a poorer prognosis with elevated serum alkaline phosphatase or lactate dehydrogenase (AP, LDH)- this is controversial

Correct Answer: CT chest and technetium bone scan

 

 

3825. (3053) Q9-3560:

A 13-year-old boy has a destructive lesion of the distal femur. Magnetic resonance images (MRIs) show a large soft tissue mass, and a needle biopsy shows a Ewing's sarcoma. Which of the following studies should be performed to complete the staging survey:

 

1) Technetium bone scan and computed tomography (CT) chest

3) Bone marrow biopsy, CT chest, and technetium bone scan

2) Technetium bone scan and CT chest and abdomen

5) CT chest and abdomen and skeletal survey

4) Skeletal survey, CT chest and abdomen, and bone marrow biopsy

 

Staging of most malignant bone tumors:

 

 

Plain radiographs and cross-sectional imaging (MRI and/or CT) Technetium bone scan - look for bone metastases

 

CT chest - look for pulmonary metastases

 

Note: In Ewing's sarcoma, bone marrow biopsy is performed to detect bone marrow metastases. Correct Answer: Bone marrow biopsy, CT chest, and technetium bone scan

A 15-year-old boy has a destructive lesion in his proximal tibia. The magnetic resonance image shows soft tissue extension and a computed tomography scan of the chest shows no evidence of pulmonary metastases. The technetium bone scan shows a solitary lesion, and a needle biopsy shows a high-grade osteosarcoma. Which of the following is the correct surgical stage:

 

1) Stage IA

3) Stage IIA

2) Stage IB

5) Stage III

IA

G1

T1

M0

IB

G1

T2

M0

IIA

G2

T1

M0

IIB

G2

T2

M0

IIIA

G1-G2

T1-T2

M1

IIIB

G1-G2

T1-T2

M1

 

4) Stage IIB Enneking System

 

Abbreviations: G=grade (G1 low: parosteal osteosarcoma, adamantinomma, chordoma; G2 high: osteosarcoma, Ewing's, malignant fibrous histiocytoma, dedifferentiated chondrosarcoma), T=site (T1: intracompartmental, T2: extracompartmental), M=metastases (M0: no metastases, M1: regional or distant).

 

Correct Answer: Stage IIB

 

 

3827. (3055) Q9-3562:

A 24-year-old man has a 1-year history of pain in his tibia. Plain radiographs show a lesion in the tibia with multiple lucent regions and intervening sclerosis. A computed tomography scan of the chest is negative, and a technetium bone scan shows a solitary lesion. The magnetic resonance image does not show any soft tissue extension, and a needle biopsy is consistent with an adamantinoma. Which of the following is the correct surgical stage:

 

1) Stage IA

3) Stage IIA

2) Stage IB

5) Stage IIII

4) Stage IIB

 

To answer this question, one must know that all adamantinomas are low-grade lesions. The suffix A is used when the tumor is intracompartmental (no soft tissue extension)

Enneking System

 

IA

G1

T1

M0

IB

G1

T2

M0

IIA

G2

T1

M0

IIB

G2

T2

M0

IIIA

G1-G2

T1-T2

M1

IIIB

G1-G2

T1-T2

M1

 

Abbreviations: G=grade (G1 low: parosteal osteosarcoma, adamantinomma, chordoma; G2 high: osteosarcoma, Ewing's, malignant fibrous histiocytoma, dedifferentiated chondrosarcoma), T=site (T1: intracompartmental, T2: extracompartmental), M=metastases (M0: no metastases, M1: regional or distant).

 

Correct Answer: Stage IA

 

Which of the following locations is the most common for osteosarcomas:

 

1) Proximal humerus

3) Distal femur

2) Proximal femur

5) Proximal tibia

4) Distal tibia

 

The distal femur is the most common location for an osteosarcoma.

 

 

Some tumors are prone to occur in certain locations. The following tumors have a common location: Osteosarcoma - knee (distal femur, proximal tibia)

 

 

Parosteal osteosarcoma - distal femur (posterior cortex) Chondrosarcoma - pelvis

 

 

 

Giant cell tumors - knee Chordoma - sacrum Adamantinoma - tibia

 

Correct Answer: Distal femur

 

 

3829. (3057) Q9-3564:

Which of the following is the most common location of giant cell tumor:

 

1) Distal radius

3) Proximal tibia

2) Proximal humerus

5) Sacrum

4) Distal femur

 

The knee is the most common location for a giant cell tumor.

 

 

Some tumors are prone to occur in certain locations. The following tumors have a common location: Osteosarcoma - knee (distal femur, proximal tibia)

 

 

Parosteal osteosarcoma - distal femur (posterior cortex) Chondrosarcoma - pelvis

 

 

 

Giant cell tumors - knee Chordoma - sacrum Adamantinoma - tibia

 

Correct Answer: Distal femur

 

 

3830. (3058) Q9-3565:

Which of the following is the most common location of chondrosarcoma of bone:

 

1) Proximal femur

3) Pelvis

2) Distal femur

5) Proximal tibia

4) Proximal humerus

 

The most common location of a chondrosarcoma of bone is the pelvis.

 

 

Some tumors are prone to occur in certain locations. The following tumors have a common location: Osteosarcoma - knee (distal femur, proximal tibia)

 

 

Parosteal osteosarcoma - distal femur (posterior cortex) Chondrosarcoma - pelvis

 

 

 

Giant cell tumors - knee Chordoma - sacrum Adamantinoma - tibia

 

Correct Answer: Pelvis

 

Patients with high-grade intramedullary osteosarcoma are best treated with which of the following therapies:

 

1) Wide resection and chemotherapy

3) Wide resection, chemotherapy, and external beam irradiation

2) Wide resection, chemotherapy, and diphosphonates

5) Wide resection alone

4) Wide resection and external beam irradiation

 

Patients with high-grade intramedullary osteosarcoma are best treated with preoperative chemotherapy, wide surgical resection, and a variable duration of postoperative chemotherapy. External beam irradiation is not useful for treating patients with intramedullary osteosarcoma.

 

The most important prognostic factor is response to preoperative chemotherapy. When necrosis rates >95% are achieved, a more favorable prognosis can be expected.

Correct Answer: Wide resection and chemotherapy

 

 

3832. (3060) Q9-3567:

A 15-year-old boy has a Ewing's sarcoma of the proximal fibula. His staging studies are negative. A moderate-sized soft tissue mass is located circumferentially around the fibula. The soft tissue mass does not involve the neurovascular bundles. Which of the following treatment alternatives would be most appropriate:

 

1) Wide resection alone

3) Chemotherapy and wide resection

2) Above knee amputation alone

5) External beam irradiation alone

4) Wide resection followed by external beam irradiation

 

Ewing's sarcoma has a high propensity to metastasize to the lungs and bone. Without intensive chemotherapy, the 5-year survival is <25%. Patients are treated with systemic chemotherapy to control micro metastases and either external beam irradiation to the local site or wide surgical resection.

 

Wide surgical resection is often used for the sites that will not result in major morbidity. If the surgical margins are positive, then patients undergo external beam irradiation.

Correct Answer: Chemotherapy and wide resection

 

 

3833. (3061) Q9-3568:

A 25-year-old woman has a parosteal osteosarcoma of the distal femur. Which of the following treatments is the most appropriate:

 

1) Wide resection alone

3) Preoperative chemotherapy, wide resection, and external beam irradiation

2) Preoperative chemotherapy and wide resection

5) Curettage and grafting

4) Wide resection and external beam irradiation

 

Parosteal osteosarcomas are low-grade osteosarcomas. The risk of pulmonary metastasis is approximately 5%. Patients are treated with wide surgical resection alone. Chemotherapy and external beam irradiation are not useful for patients with parosteal osteosarcomas.

 

Correct Answer: Wide resection alone

 

A 27-year-old man has a 1-year history of knee pain. A needle biopsy of the tibia shows an adamantinoma. Which of the following treatment options is most appropriate:

 

1) Wide resection alone

3) Wide resection and high-dose chemotherapy

2) Wide resection and low-dose chemotherapy

5) Curettage and bone grafting

4) Wide resection and external beam irradiation

 

Adamantinomas are low-grade malignancies. Adamantinomas have the potential to metastasize to the lungs, lymph nodes, and other bones. The risk of metastasis is low.

Adamantinomas are treated with wide surgical resection alone. Chemotherapy and external beam irradiation are not useful for patients with adamantinoma.

Correct Answer: Wide resection alone

 

 

3835. (3063) Q9-3570:

Which of the following treatment options is the most appropriate for a chordoma of the sacrum:

 

1) Wide resection alone

3) Wide resection and external beam irradiation

2) Wide resection and chemotherapy

5) Curettage and external beam irradiation

4) Curettage and diphosphonate therapy

 

Chordomas are low-grade malignant tumors that are best treated by wide resection. They can metastasize after a prolonged course of treatment with multiple local recurrences. Chordomas are difficult to remove, and there is a very high local failure rate.

Correct Answer: Wide resection alone

 

 

3836. (3064) Q9-3571:

Which of the following is the most appropriate treatment for patients with periosteal osteosarcoma:

 

1) Wide resection alone

3) Wide resection and chemotherapy

2) Wide resection and external beam irradiation

5) Curettage and grafting

4) Wide resection, chemotherapy, and external beam irradiation

 

Periosteal osteosarcomas are intermediate-grade malignancies. The risk of pulmonary metastases is approximately 15% to 30%. Patients are treated with chemotherapy and wide surgical resection. External beam irradiation is not useful for patients with periosteal osteosarcoma.

Correct Answer: Wide resection and chemotherapy

 

 

3837. (3065) Q9-3572:

Which of the following is the most appropriate treatment for patients with malignant fibrous histiocytoma of bone:

 

1) Wide resection alone

3) Wide resection and external beam irradiation

2) Wide resection and chemotherapy

5) Internal fixation and external beam irradiation

4) Curettage and bone grafting

 

Malignant fibrous histiocytoma of bone is usually a high-grade lesion with a marked propensity for both local recurrence and pulmonary metastases. Wide local resection is the only effective way to achieve local control. Chemotherapy is used when patients are healthy enough to undergo intensive regimens.

Correct Answer: Wide resection and chemotherapy

 

 

3838. (3066) Q9-3573:

Which of the following is the most appropriate treatment for patients with a grade 2 chondrosarcoma:

 

1) Wide resection alone

3) Wide resection and external beam irradiation

2) Wide resection and chemotherapy

5) Curettage and bone grafting

4) Curettage, cryotherapy, and bone grafting

 

Grade 2 chondrosarcomas have approximately a 20% to 30% risk of pulmonary metastases. In contrast to Grade 1 chondrosarcomas, they cause a greater degree of bone destruction and may extend into the soft tissues.

There is no role for chemotherapy or external beam irradiation. The role of curettage and adjuvant therapies, such as cryosurgery, is controversial for Grade 1 chondrosarcomas. Curettage should not be used for Grade 2 chondrosarcomas.

Correct Answer: Wide resection alone

 

 

3839. (3078) Q9-3586:

Which of the following factors causes osteoclast activation and bone destruction in multiple myeloma:

 

1) Osteoprotegerin (OPG)

3) Interferon

2) Transforming growth factor-beta (TGF-B)

5) Interleukin-4 (IL-4)

4) Receptor activator of nuclear factor-kB ligand (RANKL)

 

The bone destruction in multiple myeloma is caused by the release of RANKL from the osteoblast surface. The receptor activator of nuclear factor-kB ligand binds to a receptor activator of nuclear factor-kB (RANK) on the osteoclast progenitor cell surface, beginning the process of osteoclast activation and bone destruction. Osteoprotegerin is a decoy inhibitor of the system and halts osteoclast activation.

 

Factors that up regulate RANKL and down regulate OPG

 

 

Parathyroid hormone, parathyroid hormone-related protein 1,25 dihydroxy vitamin D

 

Interleukin-1beta

 

 

Tumor necrosis factor alpha Prostaglandin E2

 

Factors that down regulate RANKL and up regulate OPG IL-4

 

 

Interferon N gamma TGF-B

 

Factors that increase OPG expression Estrogen

 

TGF-B

Correct Answer: Receptor activator of nuclear factor-kB ligand (RANKL)

 

3840. (3079) Q9-3587:

Which of the following factors is most likely produced by the plasma cells in multiple myeloma to facilitate osteoclast activation:

 

1) Transforming growth factor-beta (TFG-B)

3) Macrophage inflammatory factor-1 alpha (MIP-1A)

2) Interleukin-4 (IL-4)

5) Parathyroid hormone-related protein (PTHrP)

4) Osteoprotegerin (OPG)

 

The bone destruction in multiple myeloma is caused by the release of the receptor activator of nuclear factor-kB ligand (RANKL) from the osteoblast surface. Receptor activator of nuclear factor-kB ligand binds to a receptor activator of nuclear factor-kB (RANK) on the osteoclast progenitor cell surface, beginning the process of osteoclast activation and bone destruction.

Osteoprotegerin is a decoy inhibitor of the system and halts osteoclast activation. Factors in multiple myeloma that cause osteoclast activation

 

IL-6

 

MIP-1A (acts directly on osteoclast precursors) Factors that up regulate RANKL and down regulate OPG

 

 

Parathyroid hormone, PTHrP 1,25 dihydroxy vitamin D

 

IL-1B

 

 

Tumor necrosis factor-alpha Prostaglandin E2

 

Factors that down regulate RANKL and up regulate OPG IL-4

 

 

Interferon N gamma TGF-B

Correct Answer: Macrophage inflammatory factor-1 alpha (MIP-1A)

 

 

3841. (3080) Q9-3588:

Which of the following factors is most likely produced by the plasma cells in multiple myeloma to facilitate osteoclast activation:

 

1) Transforming growth factor beta (TGF-B)

3) Interleukin 6 (IL-6)

2) Interleukin 4 (IL-4)

5) Parathyroid hormone-related protein (PTHrP)

4) Osteoprotegerin (OPG)

 

The bone destruction in multiple myeloma is caused by release of the receptor activator of nuclear-kB ligand (RANKL) from the osteoblast surface. The receptor activator of nuclear-kB ligand binds to a receptor activator of nuclear-kB (RANK) on the osteoclast progenitor cell surface, and the process of osteoclast activation and bone destruction begins. Osteoprotegerin is a decoy inhibitor of the system and halts osteoclast activation.

 

Factors in multiple myeloma that cause osteoclast activation IL-6

 

Macrophage inflammatory factor-alpha (acts directly on osteoclast precursors) Factors that up regulate RANKL and down regulate OPG

 

 

Parathyroid hormone, PTHrP 1,25 dihydroxy vitamin D

 

IL-1 beta

 

 

Tumor necrosis factor-alpha Prostaglandin E2

 

Factors that down regulate RANKL and up regulate OPG IL-4

 

 

Interferon N gamma TGF-B

Correct Answer: Interleukin 6 (IL-6)

 

 

3842. (3081) Q9-3589:

Which of the following agents inhibits bone destruction:

 

1) Receptor activator of nuclear-kB ligand (RANKL)

3) Interleukin-6 (IL-6)

2) Macrophage inflammatory factor-1alpha (MIP-1A)

5) Parathyroid hormone-related protein (PTHrP)

4) Osteoprotegerin (OPG)

 

The bone destruction in multiple myeloma is caused by the release of RANKL from the osteoblast surface. The receptor activator of nuclear-kB ligand binds to a receptor activator of nuclear-kB (RANK) on the osteoclast progenitor cell surface, beginning the process of osteoclast activation and bone destruction. Osteoprotegerin is a decoy inhibitor of the system and halts osteoclast activation.

 

Factors in multiple myeloma that cause osteoclast activation IL-6

 

MIP-1A (acts directly on osteoclast precursors) Factors that up regulate RANKL and down regulate OPG

 

PTHrP

 

 

1,25 dihydroxy vitamin D IL-1 beta

 

 

Tumor necrosis factor-alpha Prostaglandin E2

 

Factors that down regulate RANKL and up regulate OPG Interleukin-4

 

 

Interferon N gamma Transforming growth factor-beta

Correct Answer: Osteoprotegerin (OPG)

 

3843. (3082) Q9-3590:

Which of the following conditions are poor prognostic factors in multiple myeloma:

 

1) Increased serum albumin

3) Decreased bone marrow microvessels

2) Decreased serum beta 2 (B2) microglobulin

5) Increased serum alkaline phosphatase

4) Deletion of chromosome 13

 

Multiple myeloma is a malignant plasma dyscrasia characterized by monoclonal gammopathy and a constellation of bone lesions, renal failure, anemia, and hypercalcemia.

Diagnostic criteria

 

 

At least 10% of plasma cells in the bone marrow are affected Monoclonal protein in the serum or urine

 

End organ damage

 

 

 

Hypercalcemia Renal insufficiency Anemia

 

Bone lesions (lytic destruction) Poor prognostic factors

 

 

Complete deletion of chromosome 13 or its long arm Translocations, t(4;14), 4(14;16)

 

 

 

 

Increased bone marrow microvessels Increased plasma cell labeling index Increased serum B2 microglobulin Decreased serum albumin

 

Circulating plasma cells

Correct Answer: Deletion of chromosome 13

 

 

3844. (3083) Q9-3591:

Which of the following agents are poor prognostic factors in multiple myeloma:

 

1) Increased serum albumin

3) Increased serum alkaline phosphatase

2) Increased serum beta 2 (B2) microglobulin

5) Increased serum hemoglobin

4) Decreased bone marrow microvessels

 

Multiple myeloma is a malignant plasma dyscrasia that is characterized by monoclonal gammopathy and a constellation of bone lesions, renal failure, anemia, and hypercalcemia.

Diagnostic criteria

 

 

At least 10% of plasma cells in the bone marrow are affected Monoclonal protein in the serum or urine

 

End organ damage

 

 

 

Hypercalcemia Renal insufficiency Anemia

 

Bone lesions (lytic destruction) Poor prognostic factors

 

 

Complete deletion of chromosome 13 or its long arm Translocations, t(4;14), 4(14;16)

 

 

 

 

Increased bone marrow microvessels Increased plasma cell labeling index Increased serum B2 microglobulin Decreased serum albumin

 

Circulating plasma cells

Correct Answer: Increased serum beta 2 (B2) microglobulin

 

3845. (3092) Q9-3601:

Which of the following correctly describes a chordoma:

 

1) Aggressive benign neoplasm

3) Developmental anomaly

2) Aggressive reactive process

5) High-grade malignancy

4) Low-grade malignancy

 

A chordoma is a low-grade malignancy that has a high rate of local recurrence, but a low risk of metastasis. Although the 10-year survival rate is only approximately 25%, the majority of patients die from local spread rather than from metastases.

A number of lesions may occur in the sacrum:

Reactive processes:

 

 

Aneurysmal bone cyst Stress fracture

 

Benign aggressive neoplasm: Giant cell tumor

High-grade malignancies:

 

 

 

 

 

Metastases Myeloma Osteosarcoma Ewing's tumor Chondrosarcoma

 

Correct Answer: Low-grade malignancy

 

 

3846. (3093) Q9-3602:

Which of the following describes the correct treatment of a sacral chordoma:

 

1) Preoperative chemotherapy and wide resection

3) Wide resection alone

2) Wide resection and postoperative chemotherapy

5) External beam irradiation

4) Curettage and bone grafting

 

A chordoma is a low-grade malignancy that has a high rate of local recurrence, but a low risk of metastasis. Although the 10-year survival rate is only approximately 25%, the majority of patients die from local spread rather than from metastases.

Chordomas are treated by wide resection alone. There is no role for chemotherapy. External beam irradiation can be used for locally recurrent cases, but it seldom cures this condition.

Correct Answer: Wide resection alone

 

 

 

1) Clusters of epithelial cells in a fibrous background

3) Uniform population of giant cells in a mononuclear cell background

2) Sheets of plasma cells

5) Uniform population of small blue cell

  1. Large cells with vacuolated cytoplasm in a mucous background

     

    A chordoma is a low-grade malignancy that has a high rate of local recurrence, but a low risk of metastasis. Although the 10-year survival rate is only approximately 25%, the majority of patients die from local spread rather than from metastases.

    1. Cells have vacuolated cytoplasm and are in a myxoid intercellular matrix. Cords or sheets of cells are embedded in an abundant, pale extracellular matrix.

    2. Physaliferous cells are centrally located nucleus with a narrow rim of cytoplasm surrounded by vacuolated cytoplasm.

      1. Intracellular vacuoles

        1. Vary in size from small to large

      2. Sometimes present in small numbers

    3. Syncytial strands of cells

      1. Lying in a mass of mucous

      2. Cell boundaries are indistinct

         

        Immunochemistry stains show positivity for S-100, keratin, and epithelial membrane antigen (EMA. Correct Answer: Large cells with vacuolated cytoplasm in a mucous background

         

        3848. (3138) Q9-3957:

        Which of the following statements is true regarding lipoma arborescens:

         

        1) Lipoma arborescens is a relatively common condition.

        3) Microscopic appearance is not characteristic in lipoma arborescens.

        2) The synovium does not have macroscopic pathology.

        5) Symptoms of lipoma arborescens include diffuse joint swelling and repeated locking.

        4) Arthroscopy is not helpful in treating patients with lipoma arborescens.

         

        In patients with lipoma arborescens, joint swelling, repeated locking, and joint effusions are present.

         

        Only a few cases of lipoma arborescens have been reported.

         

         

        Macroscopically, the entire synovium is thickened in patients with lipoma arborescens. Histologically, the sub-synovium contains typical mature fatty tissue.

         

        The diagnosis of lipoma arborescens can be made at arthroscopy.

        Correct Answer: Symptoms of lipoma arborescens include diffuse joint swelling and repeated locking.

         

         

        3849. (3139) Q9-3958:

        The management of lipoma arborescens includes:

         

        1) Differential diagnosis includes cruciate ligament lesions and patella chondromalatia.

        3) Joint effusion can be obtained via puncture.

        2) Magnetic resonance imaging is not useful in diagnosis.

        5) Synovectomy is the treatment of choice.

        4) Lipoma arborescens often occurs in various upper limb joints.

         

        Synovectomy is an effective treatment with excellent results and a low recurrence rate.

         

         

        Lipoma arborescens should be differentiated from villonodular synovitis and synovial hemangioma. Magnetic resonance imaging enables diagnosis at an early stage.

         

        Effusion cannot be used in cases of lipoma arborescens.

         

        The primary location of lipoma arborescens is the knee joint.

        Correct Answer: Synovectomy is the treatment of choice.

         

         

         

        1) May exclusively involve synovial joint membranes

        3) Always appears localized

        2) Never involves bursae

        5) Has no connection with giant cell tumor of the tendon sheath

        4) Is classified as a benign proliferative lesion

         

        Villonodular synovitis is a group of benign proliferative lesions of the synovium and the mesenchymal supporting elements, tendon sheaths, and bursae.

         

         

        PVNS may involve the synovial lining of joints, tendon sheaths, and bursae (pigmented villonodular bursitis). PVNS can be further subdivided to a diffuse and a localized form.

         

        A form of PVNS is also called giant cell tumor of the tendon sheath.

        Correct Answer: Is classified as a benign proliferative lesion

         

         

        3851. (3141) Q9-3960:

        Pigmented villonodular synovitis (PVNS):

         

        1) Has no connection with chromosomal abnormalities

        3) Does not recur after excision

        2) Occurs mainly in elderly women

        5) Is considered an inflammation of the synovium

        4) Is considered a benign neoplasm of the synovium

         

        Based on cytogenetic studies and other histology findings, villonodular synovitis is considered a benign neoplasm of the synovium.

         

        Several chromosomal abnormalities such as trisomy 7 [40], aberration of short arm of chromosome 1 [41] and x chromosome inactivation [42] have been reported.

         

         

        PVNS affects all age groups and both male and female patients. PVNS has a propensity to recur after excision.

        Correct Answer: Is considered a benign neoplasm of the synovium

         

         

        3852. (3142) Q9-3961:

        Symptoms of pigmented villonodular synovitis (PVNS):

         

        1) Never appear on the joints of the upper limb

        3) Do not result in locking of the joint

        2) Cause no changes on the bone

        5) Always cause painless swelling

        4) Usually involves the knee

         

        The diffuse form of PVNS usually involves the knee as a monoarthropathy. Apart from the knee, this disorder has been described in the ankle, hip, wrist, spine, shoulder, and temporomandibular joint.

         

        PVNS lesions presented in giant cell tumor of the tendon sheath are the most common soft tissue tumor of the hand. The presenting symptom in this lesion is a painless mass on a finger.

         

         

         

        Ten percent to 25% of the lesions cause erosion and atrophic pressure on the underlying cortical bone. Intra-articular PVNS, a rare manifestation of the disorder, can present with pain and locking of the joint. The pedunculated nodules can undergo torsion and infarction, leading to acute pain.

         

        Correct Answer: Usually involves the knee

         

         

         

        1) Magnetic resonance imaging does not contribute to diagnosis and treatment planning.

        3) Marginal surgical excision of the giant cell tumor of the tendon sheath is the treatment of choice. In the knee joint, successful arthroscopic removal of the lesion has also been reported.

        2) Blood collection in the knee excludes PVNS.

        5) Osmic acid synovectomy is useless.

        4) The recurrence rate is high.

         

        Localized or focal forms of PVNS can be successfully treated with arthroscopic partial synovectomy. Radiation treatment is generally reserved for high-risk patients when the extent of the lesion makes it inoperable or in patients who have recurrent PVNS after surgery.

         

         

         

        Magnetic resonance imaging is especially useful for delineating the extent of the lesion to assist in surgical planning. Hemorrhage of joints is common.

         

         

        The recurrence rate is low and is dependent on the degree of technical difficulty. Adequately excised lesions do not recur. Osmic acid or radiation synovectomy is proposed when surgery cannot be performed.

        Correct Answer: Marginal surgical excision of the giant cell tumor of the tendon sheath is the treatment of choice. In the knee joint, successful arthroscopic removal of the lesion has also been reported.

         

         

        3854. (3144) Q9-3963:

        Which of the following statements is true regarding synovial chondroma:

         

        1) Synovial chondroma is frequently seen in clinics.

        3) Synovial chondroma predominantly occurs on major limb joints.

        2) Synovial chondroma may present in extensive size.

        5) Synovial chondroma grows rapidly.

        4) Synovial chondroma is exclusively found on the hands and feet.

         

        Synovial chondromas appear around the small joints of the hands in 64% of patients and of the feet in 20% of patients.

         

         

        Synovial chondroma is a rare chondroid tumor. Synovial chondroma does not grow larger than 2 cm.

         

        Synovial chondroma is a slow growing tumor; therefore, clinical symptoms are generally aspecific.

        Correct Answer: Synovial chondroma is exclusively found on the hands and feet.

         

         

        3855. (3145) Q9-3964:

        Synovial chondroma:

         

        1) Is frequently seen in childhood

        3) Must be differentiated from chondrosarcoma of the hand

        2) Has no distinct radiographic appearance

        5) Commonly recurs

        4) Is not adequately treated by marginal surgical excision

         

        Synovial chondroma must be distinguished from giant cell tumor of the tendon sheath, synovial chondromatosis, and chondrosarcoma of the hand. Chondrosarcoma grows more rapidly than synovial chondroma and involves the surrounding bones.

         

        The lesions develop at any age, although they are more frequent in the third and fourth decades.

         

        Radiographs often show a lobular appearance on the palmar or dorsal side of the small joints, with various calcifications and ossifications.

         

        Marginal surgical excision is the treatment choice, which is generally not followed by recurrence.

        Correct Answer: Must be differentiated from chondrosarcoma of the hand

         

         

         

        1) Is an atypical non-infectious inflammation

        3) May be a part of a generalized hemangiomatosis syndrome

        2) Mainly appears in young boys

        5) Is always localized

        4) Is relatively common

         

        Synovial hemangioma may be a part of a heterogenous syndrome, involving hemangiomas of the skin, bone, and muscles (Klippel-Trenaunay-Weber disease).

         

         

        Synovial hemangioma is a proliferation of blood vessels arising in a synovium-lined surface. Synovial hemangioma appears at any age and involves both men and women.

         

        Synovial hemangioma is rare.

         

        Synovial hemangioma appears in either a localized or diffuse form.

        Correct Answer: May be a part of a generalized hemangiomatosis syndrome

         

         

        3857. (3147) Q9-3966:

        In patients with synovial hemangioma:

         

        1) Severe pain of the involved joint is characteristic

        3) Surgical excision is ineffective

        2) Hemarthrosis of different origin is rare

        5) Histology is not distinctive

        4) Arthroscopy may reveal a pathognomic peduncular purple-blue tumor

         

        During arthroscopy, occasionally a peduncular, purple-blue tumor is pathognomic.

         

        Clinical symptoms are mild in patients with synovial hemangioma.

         

        In the differential diagnoses, pigmented villonodular synovitis, hemarthrosis of traumatic origin, meniscal lesions, and other causes of hemarthrosis should be considered.

         

        Surgical excision is indicated, if possible, at an early stage before the slowly progressing lesion infiltrates the articular synovium.

         

        The histology shows hemangioma of capillary or cavernous type.

        Correct Answer: Arthroscopy may reveal a pathognomic peduncular purple-blue tumor

         

         

        3858. (3148) Q9-3967:

        Which of the following statements is true regarding synovial osteochondromatosis:

         

        1) Primary synovial chondromatosis is a rare tumor-like lesion.

        3) Synovial chondromatosis is a poorly delineated histological entity.

        2) Synovial osteochondromatosis is based exclusively on cartilaginous metaplasia.

        5) Etiology is clarified.

        4) Subsynovial metaplastic chondroid islands can be misleading.

         

        Primary synovial chondromatosis is an uncommon tumor-like lesion.

         

         

        Synovial osteochondromatosis is based on cartilaginous metaplasia, originating from the synovium. The condition is not a simple metaplasia but has a proliferative component, as well.

         

         

        Synovial chondromatosis is a histologically verified entity with subsynovial metaplastic chondroid islands. The etiology of the process is unknown.

        Correct Answer: Primary synovial chondromatosis is a rare tumor-like lesion.

         

         

         

        1) Synovial osteochondromatosis occurs mainly in women.

        3) Synovial osteochondromatosis usually develops polyarticularly.

        2) Synovial osteochondromatosis occurs predominantly in children.

        5) Range of motion is always limited in patients with synovial osteochondromatosis.

        4) Synovial osteochondromatosis is characterized by swelling.

         

        The leading symptom in synovial osteochondromatosis is swelling of the affected joint.

         

         

         

        Found equally in men and women Occurs between 30 and 50 years of age Condition is rarely polyarticular

         

        May develop without limitation of movement

        Correct Answer: Synovial osteochondromatosis is characterized by swelling.

         

         

        3860. (3150) Q9-3969:

        Regarding synovial osteochondromatosis imaging:

         

        1) Presents as narrowing the affected joint space on radiograph

        3) Widening of the articular joint space is an indirect radiographic sign

        2) Even at the early stage, calcified loose bodies of various sizes show on radiographs

        5) Calcified loose bodies are uncommon in any other entity

        4) Magnetic resonance imaging and computed tomography are not helpful at the early stage

         

        Radiographs show widening of the articular joint space due to joint effusion.

         

        Calcified loose bodies appear only at later stage

         

         

        Magnetic resonance imaging and computed tomography detect chondromas at early stage Calcified loose bodies are also present in osteochondritis dissecans and advanced osteoarthritis

        Correct Answer: Widening of the articular joint space is an indirect radiographic sign

         

         

        3861. (3151) Q9-3970:

        Which of the following treatment options is best for synovial osteochondromatosis:

         

        1) Nonsteroidal anti-inflammatory drugs

        3) Open arthrotomy and total synovectomy

        2) Arthroscopic surgery to remove loose chondromas and partial synovectomy to prevent locking episodes and pain

        5) Malignant transformation often occurs

        4) Recurrence rate after surgery is more than 60%

         

        Partial synovectomy and arthroscopic surgery is usually appropriate.

         

         

         

        Nonsteroidal anti-inflammatory drugs provide only some relief of symptoms Open surgery is not necessary because the entire synovium is not involved The rate of recurrence is estimated to be 15% to 25%

         

        Synovial chondromatosis can become malignant

        Correct Answer: Arthroscopic surgery to remove loose chondromas and partial synovectomy to prevent locking episodes and pain

         

         

         

        1) Synovial sarcomas are rare malignant soft tissue tumors.

        3) A synovial sarcoma is usually located in the vicinity of spine.

        2) Convincing histological evidence shows that synovial sarcomas originate from the cells of the synovium.

        5) Synovial sarcoma can be mistaken for synovitis or a ganglion.

        4) Synovial sarcomas never cause pain.

         

        A synovial sarcoma may persist for years as a small, superficial, indolent lump in the hands or feet. It can be mistaken for synovitis or a ganglion.

         

        Synovial sarcoma is the third most frequent malignant soft tissue tumor; accounting for 5% to 10% of all soft tissue tumors.

         

        This tumor was designated as a synovial sarcoma because of its anatomical location (most of them originate from the capsules of the joints, bursae, and tendon sheaths) and its histological similarity to normal synovial tissue. No convincing evidence exists that the tumor originates from the cells of the synovium.

         

         

        A synovial sarcoma is usually located in the extremities (90%) and the lower limbs (60%). Deep, radiating pain is present in about 50% of patients.

        Correct Answer: Synovial sarcoma can be mistaken for synovitis or a ganglion.

         

         

        3863. (3153) Q9-3972:

        Which of the following statements is true regarding the diagnosis of synovial sarcoma:

         

        1) Radiograph shows no specific signs.

        3) Ultrasound can easily differentiate synovial sarcoma from other benign cysts.

        2) Magnetic resonance imaging exhibits lobular structure and extent.

        5) Synovial sarcoma does not have cytogenetic abnormality.

        4) Synovial sarcoma does not infiltrate the viscera and muscles.

         

        Magnetic resonance imaging is the best tool for detection of the lesion's lobular structure and extent.

         

        The tumor may be recognized on conventional plain radiographic films because tissue calcification occurs in approximately 20% to 40% of patients.

         

        Ultrasound may be helpful in distinguishing the tumor, mainly from Baker cysts in the popliteal region, although in many patients, the synovial sarcoma may contain large multicompartmental cysts filled with mucoid substance similar to synovial fluid.

         

        In the later stages, synovial sarcoma starts to spread into the viscera and muscles, infiltrating the surrounding joints and bones.

         

        The reciprocal translocation t(X; 18) (p 11.2; q 11.2) is thought to be the primary cytogenetic abnormality and specific to synovial sarcoma. By cytogenetic examinations, tumor-specific chimeric transcripts such as SYT-SSX1, SYT-SSX2 can be revealed in the synovial sarcoma, which are also highly specific for the tumor.

         

        Correct Answer: Magnetic resonance imaging exhibits lobular structure and extent.

         

         

         

        1) The survival rate for synovial sarcoma depends on the stage of the tumor and has been estimated at 25% to 60%.

        3) Risk factors for local recurrence do not include tumor size and surgical margins.

        2) Amputation is indicated to avoid local recurrence.

        5) The localization or histological (mono- or biphasic) type of the tumor are significant prognostic factors.

        4) Risk factors for tumor-related death are unrelated to histological differentiation, tumor size, age, and development of local recurrence.

         

        The 5-year survival rate for synovial sarcoma depends on the stage of the tumor and has been estimated at 25% to 60%.

         

        The results of limb-sparing surgery are no worse than amputation if wide excision is performed.

         

        Risk factors for local recurrence include large tumor size (more than 5 cm) and inadequate surgical margins.

         

        Risk factors for tumor-related death are poor histological differentiation, large tumor size, age older than 25 years, and development of local recurrence.

         

        The localization or histological (mono- or biphasic) type of the tumor are not considered to be significant prognostic factors.

        Correct Answer: The survival rate for synovial sarcoma depends on the stage of the tumor and has been estimated at 25% to 60%.

         

         

        3865. (3302) Q9-4148:

        Which of the following factors is most likely responsible for hypercalcemia of malignancy:

         

        1) Insulin-like growth factor 1

        3) TGF beta

        2) p53 protein

        5) PTHrP

        4) IL-6

         

        PTHrP is the most likely factor that causes hypercalcemia of malignancy. PTHrP is released form the cancer cells (as in breast carcinoma). The PTHrP then causes the RANKL to be released from osteoblasts and marrow stromal cells. The RANKL then attaches to the RANK receptor on the osteoclast precursor cells. If MCS-F (macrophage colony stimulating factor) is present, the osteoclast precursor cells then differentiate into osteoclasts and resorb the cortical and trabecular bone. When the organic and inorganic components of the bone are resorbed, calcium is released.Correct Answer: PTHrP

         

         

        3866. (3303) Q9-4149:

        In which of the following tumors do p53 mutations occur:

         

        1) Neurofibromatosis and multiple hereditary exostoses

        3) Retinoblastoma and desmoid tumors

        2) Osteosarcoma and high-grade chondrosarcoma

        5) Retinoblastoma and multiple hereditary exostoses

        4) Neurofibromatosis and desmoid tumor

         

        P53 mutations occur commonly in osteosarcoma, high-grade chondrosarcomas, and soft tissue sarcomas. P53 and retinoblastoma are the two most important tumor suppressor genes.

        P53 mutations may result in a syndrome called Li-Fraumeni syndrome. Patients may develop breast cancer, osteogenic sarcoma, soft tissue sarcomas, and other malignancies.

        One should remember the major tumor suppressor genes in musculoskeletal conditions:

         

        Retinoblastoma (RB) gene â retinoblastoma, osteosarcoma

         

         

         

         

        p53 â osteosarcoma, high-grade chondrosarcoma, soft tissue sarcomas P16INK4a â familial melanoma â chondrosarcoma, osteosarcoma, melanoma APC â familial adenomatous polyposis â colon adenomas, desmoid tumors NF1 â neurofibromatosis 1 â neurofibroma, sarcoma

         

         

        EXT1, EXT2 â hereditary multiple exostosis â osteochondromas, chondrosarcoma Correct Answer: Osteosarcoma and high-grade chondrosarcoma

        retinoblastoma (RB) gene:

         

        1) Chondrosarcoma

        3) Neurofibrosarcoma

        2) Malignant fibrous histiocytoma

        5) Breast cancer

        4) Osteosarcoma

         

        Patients who have mutation in the retinoblastoma gene are prone to develop retinoblastoma. Osteosarcoma is the second most common malignancy to develop in these patients.

        One should remember the major tumor suppressor genes in musculoskeletal conditions:

         

        Retinoblastoma (RB) gene â retinoblastoma, osteosarcoma

         

         

         

         

        p53 â osteosarcoma, high-grade chondrosarcoma, soft tissue sarcomas P16INK4a â familial melanoma â chondrosarcoma, osteosarcoma, melanoma APC â familial adenomatous polyposis â colon adenomas, desmoid tumors NF1 â neurofibromatosis 1 â neurofibroma, sarcoma

         

         

        EXT1, EXT2 â hereditary multiple exostosis â osteochondromas, chondrosarcoma Correct Answer: Osteosarcoma

         

        3868. (3305) Q9-4151:

        Which of the following blocks the restriction checkpoint in the cell cycle (prohibits cells from passing from G1 to the S [synthesis] phase):

         

        1) E2F

        3) Cyclin-dependent kinases

        2) RB gene

        5) p53

        4) INK (inhibitor of CDK4)

         

        The retinoblastoma (RB) gene controls the checkpoint in regard to cells passing from the G1 (or G0 resting phase) into S, or synthesis, phase.

        One should remember that the RB gene is phosphorylated by cyclin-dependent kinases (CDKs). When the RB protein is phosphorylated, it dissociates from E2f, which is a transcription factor for a number of enzymes for DNA synthesis.

         

        The CDK inhibitors block the cell from progressing through the synthesis phase. These inhibitors include: p21WAF/Cip1, p27Kip1, p57Kip2

         

        INK

        The important point is to remember that the retinoblastoma gene is a tumor suppressor gene and it controls the restriction point of the cell cycle.

        Correct Answer: RB gene

         

         

         

        1) IL-6-mediated inflammation

        3) PTHrP-mediated bone resorption

        2) TNF-mediated inflammation

        5) E2F transcription factor activation of enzymes for DNA synthesis

        4) Programmed cell death

         

        Apoptosis is programmed cell death. This process is characterized by nuclear condensation, DNA fragmentation, and cell shrinkage. The cysteine proteases play an important role for enzymatic degradation.

        One should remember that chemotherapy works in general by inducing programmed cell death (or apoptosis). The various agents damage the DNA, which causes induction of the apoptic pathways.

        Correct Answer: Programmed cell death

         

         

        3870. (3307) Q9-4157:

        Which of the following may account for the predisposition of breast carcinoma cells to metastasize to bone:

         

        1) PTHrP

        3) Integrins

        2) RANKL

        5) E-cadherin

        4) p53

         

        Integrins are membrane receptors that heavily influence cell migration and attachment. They recognize the RGD (Arg-Gly-Asp) peptide on extracellular matrix molecules. The role of integrins may be important in breast carcinoma metastasis to bone.

        Cadherins are cell adhesion molecules. When breast carcinoma cells lose E-cadherins, there may be an enhanced ability to metastasize.

        Correct Answer: Integrins

         

         

        3871. (3308) Q9-4158:

        Which of the following has been found in chondrosarcomas and may indicate the degree of malignancy:

         

        1) RB

        3) MMPs (matrix metalloproteinases)

        2) p53

        5) PTHrP

        4) TIMPs

         

        Matrix metalloproteinases (MMPs) may play an important role in the degree of malignancy of chondrosarcomas. These enzymes have the ability to degrade the extra-cellular matrix. Matrix metalloproteinases include collagenases, gelatinases, and stromelysins.

         

        Tissue inhibitors of metalloproteinases (TIMPs) act to hold these derivative enzymes in check to prevent unwanted destruction. Correct Answer: MMPs (matrix metalloproteinases)

         

         

        1) Size >5 cm

        3) SYT-SSX1 fusion product

        2) Location below the fascia

        5) Single regional lymph node metastasis

        4) SYT-SSX2 fusion product

         

        The SYT-SSX1 fusion product is associated with a better prognosis than the SYT-SSX2 gene fusion. All of the other responses portend a poor prognosis.

        Poor prognostic factors in synovial sarcoma:

         

         

        Presence of lymph node or pulmonary metastasis Size >5 cm

         

        Location below the fascia

         

         

        gene fusion product SYT-SSX2 Correct Answer: SYT-SSX1 fusion product

         

        3873. (3310) Q9-4162:

        Which of the following is the major effect of chemotherapy in the treatment of osteosarcoma:

         

        1) Improves local control

        3) Reduces the risk for brain metastases

        2) Reduces the risk for bone metastases

        5) Reduces the risk for skin metastases

        4) Reduces the risk for pulmonary metastases

         

        The major effect of chemotherapy in treating patients with osteosarcoma is reducing the risk of pulmonary metastases. Prior to the advent of multi-agent chemotherapy, the survival of children with osteosarcoma was only 10% to 20% because of pulmonary metastases. With current preoperative and postoperative chemotherapy regimens, the survival has improved to 70%.Correct Answer: Reduces the risk for pulmonary metastases

         

         

        3874. (3311) Q9-4163:

        Which of the following controls the transition from the G1 to S phase by recognizing DNA damage and halting cell cycle progression:

         

        1) Retinoblastoma protein

        3) E2F

        2) Cyclin D

        5) p53 protein

        4) CDK4

         

        The p53 protein is referred to as the âguardian of the genome.â The p53 protein detects DNA damage and halts progression of the cell cycle (synthesis of DNA and replication). This protein either halts synthesis, allowing DNA repair, or, if the cell is past the restriction point, p53 induces programmed cell death (apoptosis).

         

        If DNA damage occurs, then the p53 protein is induced. The increase in p53 causes an increase in p21, which prevents phosphorylation of the retinoblastoma protein. One should remember that phosphorylation of the retinoblastoma protein leads to the transcription of various factors that are necessary for DNA synthesis.

         

        Correct Answer: p53 protein

         

         

         

        1) Receptor activator of nuclear factor-kB ligand/receptor activator nuclear factor-kB

        3) Transforming growth factor beta/insulin-like growth factor 1

        2) Receptor activator of nuclear factor-kB ligand/osteoprotegerin

        5) Tumor necrosis factor

        4) Retinoblastoma

         

        The retinoblastoma and p53 pathways are the two major regulators of the cell cycle progression from the resting phase to the S or synthesis phase. One should remember that phosphorylation of the retinoblastoma protein results in release of transcription factors, whereas the p53 pathway detects genetic damage and halts phosphorylation of the retinoblastoma protein.

         

         

        Both of these pathways must be damaged to have progression of tumor cells through cell growth. Correct Answer: Retinoblastoma

         

        3876. (3313) Q9-4165:

        Which of the following pathways must be inactivated for tumor cell growth:

         

        1) Receptor activator of nuclear factor-kB ligand/receptor activator nuclear factor-kB

        3) Transforming growth factor beta/insulin-like growth factor 1

        2) Receptor activator of nuclear factor-kB ligand/osteoprotegerin

        5) Tumor necrosis factor/interleukin 6

        4) p53

         

        The retinoblastoma and p53 pathways are the two major regulators of the cell cycle progression from the resting phase to the S or synthesis phase. One should remember that phosphorylation of the retinoblastoma protein results in release of transcription factors, whereas the p53 pathway detects genetic damage and halts phosphorylation of the retinoblastoma protein.

         

         

        Both of these pathways must be damaged to have progression of tumor cells through cell growth. Correct Answer: p53

         

        3877. (3314) Q9-4167:

        In which of the following cancers has chemotherapy shown an improvement in survival:

         

        1) Sarcoma occurring in Pagetâs disease

        3) Parosteal osteosarcoma

        2) Dedifferentiated chondrosarcoma

        5) High-grade intramedullary chondrosarcoma

        4) High-grade intramedullary osteosarcoma

         

        Chemotherapy has dramatically improved the survival rate of patients with high-grade intramedullary osteosarcoma and Ewingâs tumor. The disease-free survival rate without chemotherapy is less than 20% for patients, whereas with modern multi-agent regimens, the disease-free survival rate approaches 70%. Periosteal osteosarcoma is an intermediate-grade osteosarcoma (risk of pulmonary metastases is 15% to 25%), which is generally treated with the same chemotherapy regimen as high-grade intramedullary osteosarcoma.

         

        Chemotherapy has been used for a number of high-grade sarcomas with disappointing results: Dedifferentiated chondrosarcoma

         

         

        High-grade intramedullary chondrosarcoma Sarcoma following Pagetâs disease

        Chemotherapy has no use with low-grade sarcomas:

         

         

        Well-differentiated intramedullary osteosarcoma Parosteal osteosarcoma

         

         

        Low-grade intramedullary chondrosarcoma Adamantinoma

         

        Chordoma

        Correct Answer: High-grade intramedullary osteosarcoma

         

        3878. (3315) Q9-4168:

        In which of the following cancers has chemotherapy shown an improvement in survival:

         

        1) Dedifferentiated chondrosarcoma

        3) Ewingâs tumor

        2) Sarcoma complicating Pagetâs disease

        5) High-grade intramedullary chondrosarcoma

        4) Parosteal osteosarcoma

         

        Chemotherapy has dramatically improved the survival rate of patients with high-grade intramedullary osteosarcoma and Ewingâs tumor. The disease-free survival rate without chemotherapy is less than 20% for patients, whereas with modern multi-agent regimens, the disease-free survival rate approaches 70%. Periosteal osteosarcoma is an intermediate-grade osteosarcoma (risk of pulmonary metastases is 15% to 25%), which is generally treated with the same chemotherapy regimen as high-grade intramedullary osteosarcoma.

         

        Chemotherapy has been used for a number of high-grade sarcomas with disappointing results: Dedifferentiated chondrosarcoma

         

         

        High-grade intramedullary chondrosarcoma Sarcoma following Pagetâs disease

        Chemotherapy has no use with low-grade sarcomas:

         

         

        Well-differentiated intramedullary osteosarcoma Parosteal osteosarcoma

         

         

        Low-grade intramedullary chondrosarcoma Adamantinoma

         

        Chordoma

        Correct Answer: Ewingâs tumor

         

         

        3879. (3316) Q9-4169:

        Chemotherapy has no role in the treatment of which of the following tumors:

         

        1) High-grade intramedullary osteosarcoma

        3) Periosteal osteosarcoma

        2) Ewingâs tumor

        5) Multiple myeloma

        4) Parosteal osteosarcoma

         

        Chemotherapy has dramatically improved the survival rate of patients with high-grade intramedullary osteosarcoma and Ewingâs tumor. The disease-free survival rate without chemotherapy is less than 20% for patients, whereas with modern multi-agent regimens, the disease-free survival rate approaches 70%. Periosteal osteosarcoma is an intermediate-grade osteosarcoma (risk of pulmonary metastases is 15% to 25%), which is generally treated with the same chemotherapy regimen as high-grade intramedullary osteosarcoma.

         

        Chemotherapy has been used for a number of high-grade sarcomas with disappointing results: Dedifferentiated chondrosarcoma

         

         

        High-grade intramedullary chondrosarcoma Sarcoma following Pagetâs disease

        Chemotherapy has no use with low-grade sarcomas:

         

         

        Well-differentiated intramedullary osteosarcoma Parosteal osteosarcoma

         

         

        Low-grade intramedullary chondrosarcoma Adamantinoma

         

        Chordoma

        Correct Answer: Parosteal osteosarcoma

         

        Chemotherapy has no role in the treatment of which of the following tumors:

         

        1) High-grade surface osteosarcoma

        3) Adamantinoma

        2) High-grade intramedullary osteosarcoma

        5) Ewingâs tumor

        4) Periosteal osteosarcoma

         

        Chemotherapy has dramatically improved the survival rate of patients with high-grade intramedullary osteosarcoma and Ewingâs tumor. The disease-free survival rate without chemotherapy is less than 20% for patients, whereas with modern multi-agent regimens, the disease-free survival rate approaches 70%. Periosteal osteosarcoma is an intermediate-grade osteosarcoma (risk of pulmonary metastases is 15% to 25%), which is generally treated with the same chemotherapy regimen as high-grade intramedullary osteosarcoma.

         

        Chemotherapy has been used for a number of high-grade sarcomas with disappointing results: Dedifferentiated chondrosarcoma

         

         

        High-grade intramedullary chondrosarcoma Sarcoma following Pagetâs disease

        Chemotherapy has no use with low-grade sarcomas:

         

         

        Well-differentiated intramedullary osteosarcoma Parosteal osteosarcoma

         

         

        Low-grade intramedullary chondrosarcoma Adamantinoma

         

        Chordoma

        Correct Answer: Adamantinoma

         

         

        3881. (3318) Q9-4171:

        Which of the following is associated with multiple drug resistance in malignancies:

         

        1) Retinoblastoma protein

        3) p21

        2) p53

        5) Tumor necrosis factor

        4) p-glycoprotein

         

        P-glycoprotein is associated with multiple drug resistance. This complex of proteins pumps cancer treatment drugs out of the cell (efflux). This action may be accomplished by establishing an electrochemical gradient that facilitates drug efflux. This is an active area of research in which the molecular mechanisms have not been fully elucidated.

         

        Many studies show that sarcomas have variable amounts of immunostaining for p-glycoprotein. Correct Answer: p-glycoprotein

        A 10-year-old boy presents with rapidly growing soft tissue masses. The soft tissue masses mineralize over a 2-month period. He has short great toes with hallux valgus deformity. The most likely diagnosis is:

         

        1) Polyostotic fibrous dysplasia

        3) Cleidocranial dysplasia

        2) Achondroplasia

        5) Multifocal osteosarcoma

        4) Fibrodysplasia ossificans progressiva

         

         

        Clinical presentation of fibrodysplasia (myositis) ossificans progressiva: Painful soft tissue masses (rapid progression)

         

         

        Spontaneously Minor trauma

         

         

        Injections Falls

         

        Location

         

        Paraspinal muscles, extremities

         

         

        Aponeuroses, fascia, tendons, ligaments, voluntary skeletal muscle Contractures

         

         

        Ossification of muscle bellies results in contractures/ankylosis Hips

         

         

        Spine Systemic problems

         

         

        Jaw ankylosis â unable to eat Scoliosis â restrictive lung disease

        Pathophysiology:

         

        Perivascular infiltration of B, T lymphocytes

         

         

        Progresses to pure T lymphocyte infiltration of the muscles Enchondral ossification occurs

         

         

        Intense immunostaining for BMP 2/4 Correct Answer: Fibrodysplasia ossificans progressiva

         

        slide 1 slide 2 slide 3

        A patient presents with a hard leg mass and pain with activity. The anteroposterior and lateral radiographs are shown in Slide 1 and Slide 2. An axial computed tomography scan is shown in Slide 3. Which of the following is the most likely inheritance pattern:

         

        1) X-linked recessive

        3) Autosomal recessive

        2) X-linked dominant

        5) Sporadic

        4) Autosomal dominant

         

        This patient has multiple hereditary exostoses. Widening of the metaphysis is characteristic of multiple hereditary exostoses. Large sessile osteochondromas arise from the metaphysis and a large osteochondroma arises from the medial metaphysis with a characteristic cartilaginous cap. The computed tomography scan shows the widening and abnormal tubulation of the bone.

         

        No evidence of malignancy exists in this large osteochondroma. The cartilage cap is regular with no areas of bone destruction. One should also look for a soft tissue mass, which often shows areas of focal calcifications. No soft tissue masses are present in this patient.

         

        It is important to remember that this condition is autosomal dominant. The putative tumor suppressive gene mutation is EXT1, EXT2. The risk of low-grade chondrosarcoma occurring in this condition is approximately 10%.

        Correct Answer: Autosomal dominant

         

         

        slide 1 slide 2 slide 3

        A patient presents with a hard leg mass and pain with activity. The anteroposterior and lateral radiographs are shown in Slide 1 and Slide 2. An axial computed tomography scan is shown in Slide 3. Which of the following tumor suppressor genes is most likely involved:

         

        1) Retinoblastoma (RB)

        3) P16INK4a

        2) p53

        5) NF1

        4) EXT1

         

        This patient has multiple hereditary exostoses. Widening of the metaphysis is characteristic of multiple hereditary exostoses. Large sessile osteochondromas arise from the metaphysis and a large osteochondroma arises from the medial metaphysis with a characteristic cartilaginous cap. The computed tomography scan shows the widening and abnormal tubulation of the bone.

         

        No evidence of malignancy exists in this large osteochondroma. The cartilage cap is regular with no areas of bone destruction. One should also look for a soft tissue mass, which often shows areas of focal calcifications. No soft tissue masses are present in this patient.

         

        It is important to remember that this condition is autosomal dominant. The putative tumor suppressive gene mutation is EXT1, EXT2. The risk of low-grade chondrosarcoma occurring in this condition is approximately 10%.

        Correct Answer: EXT1

         

         

        slide 1 slide 2 slide 3

        A patient presents with a hard leg mass and pain with activity. The anteroposterior and lateral radiographs are shown in Slide 1 and Slide 2. An axial computed tomography scan is shown in Slide 3. Which of the following would be the most appropriate treatment:

         

        1) Simple excision

        3) Preoperative chemotherapy and wide resection

        2) Wide resection of the distal femur

        5) Above-knee amputation

        4) Wide resection and external beam irradiation

         

        This patient has multiple hereditary exostoses. Widening of the metaphysis is characteristic of multiple hereditary exostoses. Large sessile osteochondromas arise from the metaphysis and a large osteochondroma arises from the medial metaphysis with a characteristic cartilaginous cap. The computed tomography scan shows the widening and abnormal tubulation of the bone.

         

        No evidence of malignancy exists in this large osteochondroma. The cartilage cap is regular with no areas of bone destruction. One should also look for a soft tissue mass, which often shows areas of focal calcifications. No soft tissue masses are present in this patient.

         

        It is important to remember that this condition is autosomal dominant. The putative tumor suppressive gene mutation is EXT1, EXT2. The risk of low-grade chondrosarcoma occurring in this condition is approximately 10%.

         

        When patients are symptomatic, simple excision of the osteochondroma is all that is necessary. Correct Answer: Simple excision

         

        slide 1 slide 2 slide 3

        A patient presents with a hard leg mass and pain with activity. The anteroposterior and lateral radiographs are shown in Slide 1 and Slide 2. An axial computed tomography scan is shown in Slide 3. The risk of malignancy in this condition is approximately:

         

        1. No risk of malignancy exists.

          3) 25%

        2. 5% to 10%

  2. 100%

4) 50%

 

This patient has multiple hereditary exostoses. Widening of the metaphysis is characteristic of multiple hereditary exostoses. Large sessile osteochondromas arise from the metaphysis and a large osteochondroma arises from the medial metaphysis with a characteristic cartilaginous cap. The computed tomography scan shows the widening and abnormal tubulation of the bone.

 

No evidence of malignancy exists in this large osteochondroma. The cartilage cap is regular with no areas of bone destruction. One should also look for a soft tissue mass, which often shows areas of focal calcifications. No soft tissue masses are present in this patient.

 

It is important to remember that this condition is autosomal dominant. The putative tumor suppressive gene mutation is EXT1, EXT2. The risk of low-grade chondrosarcoma occurring in this condition is approximately 10%.

 

When patients are symptomatic, simple excision of the osteochondroma is all that is necessary. Correct Answer: 5% to 10%

 

3887. (3324) Q9-4179:

Which of the following statements is true concerning total knee replacement in patients with pigmented villonodular synovitis (PVNS) of the knee:

 

  1. Aseptic loosening secondary to recurrent disease is common.

3) Postoperative stiffness is unlikely to occur.

2) Recurrence rates are higher than in patients treated with synovectomy alone.

5) Recurrence rates are low.

4) Postoperative results are acceptable with good knee and functional scores.

 

Total knee replacement is successful in patients who have PVNS. The results are equal to total knee arthroplasty in patients in the general population and patients who have focal or inactive PVNS. Loosening and recurrence are potential complications in a patient who has active diffuse PVNS and undergoes total knee replacement.

 

Important points to remember:

 

 

Interval to arthroplasty, average 8 years (5 months to 23 years) Nine patients had 19 prior procedures

 

Six patients had 14 prior recurrences

 

 

Results (11 patients with active, diffuse disease, average age 59 years): 7/11 intact functioning knees at 11 years

 

2/7 patients required manipulation

 

3/11 required revision for aseptic loosening

 

 

Loosening was not secondary to recurrent PVNS 2/11 recurrence

 

One patient required above-knee amputation

Correct Answer: Postoperative results are acceptable with good knee and functional scores.

 

Which of the following statements is true concerning pigmented villonodular synovitis (PVNS) of the hip:

 

1) Bone and cartilage destruction occurs less commonly than in PVNS of the knee.

3) Femoral neck and acetabular cysts and erosions are uncommon.

2) Joint effusions are rare.

5) T1- and T2-weighted magnetic resonance imaging sequences often show high signal areas with prominent signal intensification.

4) Delay in diagnosis is common.

 

PVNS of the hip is difficult to diagnosis, with a long interval between the beginning of symptoms and diagnosis. Important points to remember include:

 

Hip involved in 15% of PVNS cases

 

 

Long interval from symptoms to diagnosis Hip effusion is very common

 

 

Patients present with severe hip pain and limitation of motion Nonâweight bearing cysts in >90%

 

 

Joint space narrowing in more than two-thirds Femoral neck erosions in about one-third

 

 

Bone and joint destruction is more common in PVNS of the hip compared to the knee Femoral neck and acetabular cysts are common

Correct Answer: Delay in diagnosis is common.

 

 

3889. (3326) Q9-4182:

Which of the following statements is true concerning pigmented villonodular synovitis (PVNS) of the hip:

 

1) Bone and cartilage destruction occurs more commonly than in PVNS of the knee.

3) Femoral neck and acetabular cysts and erosions are uncommon.

2) Joint effusions are rare.

5) T1- and T2-weighted magnetic resonance imaging sequences often show high signal areas with prominent signal intensification.

4) Interval between symptoms and diagnosis is short.

 

PVNS of the hip is difficult to diagnosis, with a long interval between the beginning of symptoms and diagnosis. Important points to remember include:

 

Hip involved in 15% of PVNS cases

 

 

Long interval from symptoms to diagnosis Hip effusion is very common

 

 

Patients present with severe hip pain and limitation of motion Nonâweight bearing cysts in >90%

 

 

Joint space narrowing in more than two-thirds Femoral neck erosions in about one-third

 

 

Bone and joint destruction is more common in PVNS of the hip compared to the knee Femoral neck and acetabular cysts are common

Correct Answer: Bone and cartilage destruction occurs more commonly than in PVNS of the knee.

 

Which of the following is the most likely cytotoxic effect of external beam irradiation when treating malignancies:

 

1) Damage to microvasculature

3) Depletion of cellular nutrients

2) Damage to large feeder vessels

5) Hyperthermic effect

4) Creation of free radicals

 

External beam irradiation creates free radicals. The free radicals damage the DNA. When the DNA cannot be repaired, cell death occurs.

 

 

The dose of irradiation is measured in grays or centigrays. A gray is a measure of the dose absorbed â 1 Gy equals 1 J/kg. One should remember the typical dose for delivered for certain tumors:

Correct Answer: Creation of free radicals

 

 

3891. (3367) Q9-4277:

Which of the following radiation doses is most commonly used in the treatment of metastatic bone disease:

 

1) 800 cGy, single fraction

3) 5040 cGy, 25 fractions

2) 3000 cGy, 10 fractions

5) 6600 cGy, 33 fractions

4) 6300 cGy, 35 fractions

 

External beam irradiation is commonly used for the treatment of metastatic bone disease. Pain control is achieved in 70% to 80% of patients (total pain relief in about 30%). Bone destruction is at least halted temporarily.

 

The common dosing regimens used for metastatic bone disease are: 3000 cGy in 10 fractions

 

 

2000 cGy in five fractions 800 cGy in a single fraction

Correct Answer: 3000 cGy, 10 fractions

 

 

3892. (3368) Q9-4278:

Which of the following radiation doses is most commonly used to control heterotopic ossification:

 

1) 800 cGy

3) 3000 cGy

2) 2000 cGy

5) 5200 cGy

4) 4500 cGy

 

External beam irradiation creates free radicals. The free radicals damage the DNA. When the DNA cannot be repaired, cell death occurs.

The dose of irradiation is measured in grays or centigrays. A gray is a measure of the dose absorbed â 1 Gy equals 1 J/kg. One should remember the typical dose for delivered for certain tumors/conditions:

 

 

 

When external beam irradiation is used to control heterotopic ossification, the radiation is delivered within 48 hours. Correct Answer: 800 cGy

Which of the following is the most significant effect of adjuvant external beam irradiation in the treatment of soft tissue sarcomas:

 

1) Improved disease-free survival

3) Improved disease-free survival and local control

2) Improved local control

5) Decreased local control, decreased disease-free survival

4) Improved local control, decreased disease-free survival

 

External beam irradiation when combined with wide surgical resection significantly decreased the risk of local recurrence in the treatment of soft tissue sarcomas. The use of preoperative or postoperative external beam irradiation does not affect disease survival.Correct Answer: Improved local control

 

 

3894. (3370) Q9-4281:

Which of the following is a late effect of external beam irradiation when used to treat a soft tissue sarcoma:

 

1) Delayed wound healing

3) Fatigue

2) Desquamation

5) Stress fractures

4) Hyperpigmentation

 

Preoperative radiation is commonly used in the treatment of soft tissue sarcomas. A frequently used regimen is 5040 cGy in 25 fractions (Monday to Friday, 180 cGy fractions daily for 5 weeks). The early effects such as radiation dermatitis are allowed to subside and, typically, at 2 to 4 weeks, the tumor is resected. Delayed wound healing is a significant problem; it occurs in up to 30% to 50% of patients. Many centers combine surgical resection with flap coverage to minimize the risk of wound dehiscence and infection.

 

Early side effects of treatment:

 

 

Radiation dermatitis Fatigue

 

Delayed wound healing Late side effects of treatment:

 

 

 

 

 

Stress fractures Postirradiation sarcomas Osteonecrosis Fibrosis/edema Neuropathy

 

Correct Answer: Stress fractures

 

Which of the following is an early effect of external beam irradiation when used as adjuvant therapy in the treatment of soft tissue sarcomas:

 

1) Stress fractures

3) Postirradiation sarcoma

2) Osteonecrosis

5) Fibrosis

4) Delayed wound healing

 

Preoperative radiation is commonly used in the treatment of soft tissue sarcomas. A frequently used regimen is 5040 cGy in 25 fractions (Monday to Friday, 180 cGy fractions daily for 5 weeks). The early effects such as radiation dermatitis are allowed to subside and, typically, at 2 to 4 weeks, the tumor is resected. Delayed wound healing is a significant problem; it occurs in up to 30% to 50% of patients. Many centers combine surgical resection with flap coverage to minimize the risk of wound dehiscence and infection.

 

Early side effects of treatment:

 

 

Radiation dermatitis Fatigue

 

Delayed wound healing Late side effects of treatment:

 

 

 

 

 

Stress fractures Postirradiation sarcomas Osteonecrosis Fibrosis/edema Neuropathy

 

Correct Answer: Delayed wound healing

 

 

3896. (3372) Q9-4284:

External beam irradiation is an effective modality for all of the following malignancies except:

 

1) Ewingâs tumor

3) Osteosarcoma

2) Lymphoma

5) Solitary myeloma of bone

4) Multiple myeloma

 

 

External beam irradiation can be effective for several malignancies such as: Ewingâs tumor

 

 

Lymphoma Myeloma

 

 

Metastatic bone disease Soft tissue sarcomas

Correct Answer: Osteosarcoma

 

Which of the following bone malignancies can be effectively managed with external beam irradiation:

 

1) Osteosarcoma

3) Lymphoma

2) Chondrosarcoma

5) Malignant fibrous histiocytoma

4) Periosteal osteosarcoma

 

 

External beam irradiation can be effective for several malignancies such as: Ewingâs tumor

 

 

Lymphoma Myeloma

 

 

Metastatic bone disease Soft tissue sarcomas

 

 

 

 

 

 

 

 

 

Correct Answer: Lymphoma 3898. (3374) Q9-4286:

 

slide 1 slide 2 slide 3 slide 4

A 55-year-old woman has a 15-year history of chronic knee pain. Her plain radiographs are shown in Slide 1. Axial and sagittal T1-weighted magnetic resonance scans are shown in Slide 2. A coronal and sagittal computed tomography reconstruction is shown in Slide 3 and a biopsy in Slide 4. The most likely diagnosis is:

 

1) Synovial sarcoma

3) Rheumatoid arthritis

2) Epithelioid sarcoma

5) Septic arthritis

4) Pigmented villonodular synovitis

 

This patient has pigmented villonodular synovitis (PVNS). Patients often present with a long history of knee pain and recurrent atraumatic effusions.

The plain radiographs show large lucent areas on both sides of the knee joint. The lateral radiograph shows a large posterior erosion. The computed tomography reconstructions show large areas on bone lysis and erosions. Notice the prominent erosions on both sides of the joint. The magnetic resonance image shows large low signal masses and major areas of erosion.

 

The biopsy slide shows the typical histologic features of PVNS. Note that the macrophages have ingested hemosiderin pigment giving them a characteristic granular brown appearance.

Treatment for this patient is synovectomy and total knee replacement. The amount of bone and cartilage destruction is prominent; therefore, synovectomy alone is not sufficient.

Correct Answer: Pigmented villonodular synovitis

 

 

slide 1 slide 2 slide 3 slide 4

A 55-year-old woman has a 15-year history of chronic knee pain. Her plain radiographs are shown in Slide 1. Axial and sagittal T1-weighted magnetic resonance scans are shown in Slide 2. A coronal and sagittal computed tomography reconstruction is shown in Slide 3 and a biopsy in Slide 4. Which of the following is the most likely recommended treatment:

 

1) Arthroscopic debridement and short-term antibiotics

3) Synovectomy and total knee replacement

2) Preoperative radiation and wide resection

5) Open debridement and long-term antibiotics

4) Arthroscopic debridement and long-term antibiotics

 

This patient has pigmented villonodular synovitis (PVNS). Patients often present with a long history of knee pain and recurrent atraumatic effusions.

The plain radiographs show large lucent areas on both sides of the knee joint. The lateral radiograph shows a large posterior erosion. The computed tomography reconstructions show large areas on bone lysis and erosions. Notice the prominent erosions on both sides of the joint. The magnetic resonance image shows large low signal masses and major areas of erosion.

 

The biopsy slide shows the typical histologic features of PVNS. Note that the macrophages have ingested hemosiderin pigment giving them a characteristic granular brown appearance.

Treatment for this patient is synovectomy and total knee replacement. The amount of bone and cartilage destruction is prominent; therefore, synovectomy alone is not sufficient.

Correct Answer: Synovectomy and total knee replacement

 

 

slide 1 slide 2 slide 3 slide 4

A 55-year-old woman has a 15-year history of chronic knee pain. Her plain radiographs are shown in Slide 1. Axial and sagittal T1-weighted magnetic resonance scans are shown in Slide 2. A coronal and sagittal computed tomography reconstruction is shown in Slide 3 and a biopsy in Slide 4. The most likely etiology of this condition is:

 

1) Systemic inflammatory

3) Infectious

2) Reactive

5) Malignant neoplasm

4) Benign neoplasm

 

This patient has pigmented villonodular synovitis (PVNS). Patients often present with a long history of knee pain and recurrent atraumatic effusions.

The plain radiographs show large lucent areas on both sides of the knee joint. The lateral radiograph shows a large posterior erosion. The computed tomography reconstructions show large areas on bone lysis and erosions. Notice the prominent erosions on both sides of the joint. The magnetic resonance image shows large low signal masses and major areas of erosion.

 

The biopsy slide shows the typical histologic features of PVNS. Note that the macrophages have ingested hemosiderin pigment giving them a characteristic granular brown appearance.

Treatment for this patient is synovectomy and total knee replacement. The amount of bone and cartilage destruction is prominent; therefore, synovectomy alone is not sufficient.

 

This condition is most likely a benign reactive condition rather than a true neoplasm. Correct Answer: Reactive

 

slide 1 slide 2 slide 3 slide 4

 

 

 

slide 5 slide 6

A patient presents with increasing knee pain over a 2-year period. The anteroposterior and lateral radiographs are shown in Slide 1 and Slide 2. A computed tomography scan is shown in Slide 3 and axial T2-weighted magnetic resonance images in Slide 4 and Slide 5. A biopsy is shown in Slide 6. The most likely diagnosis is:

 

1) Synovial chondromatosis

3) Rheumatoid arthritis

2) Pigmented villonodular synovitis

5) Tuberculosis

4) Gout

 

This patient has pigmented villonodular synovitis. Patients often present with a long history of knee pain and recurrent atraumatic effusions.

The plain radiographs are often fairly normal in cases involving the knee. The lateral radiograph may show subtle erosion. The computed tomography scan shows erosions on both sides of the proximal tibiofibular joint. The axial magnetic resonance images show the characteristic large low-signal masses.

 

Slide 6 shows the typical histologic features of pigmented villonodular synovitis. Note that the macrophages have ingested hemosiderin pigment, giving them a characteristic granular appearance.

Correct Answer: Pigmented villonodular synovitis

 

 

slide 1 slide 2 slide 3 slide 4

 

 

 

slide 5 slide 6

A patient presents with increasing knee pain over a 2-year period. The anteroposterior and lateral radiographs are shown in Slide 1 and Slide 2. A computed tomography scan is shown in Slide 3 and axial T2-weighted magnetic resonance images in Slide 4 and Slide 5. A biopsy is shown in Slide 6. Which of the following best describes this condition:

 

1) Benign neoplasm

3) Systemic inflammatory

2) Malignant neoplasm

5) Infectious

4) Reactive condition

 

This patient has pigmented villonodular synovitis. Patients often present with a long history of knee pain and recurrent atraumatic effusions.

The plain radiographs are often fairly normal in cases involving the knee. The lateral radiograph may show subtle erosion. The computed tomography scan shows erosions on both sides of the proximal tibiofibular joint. The axial magnetic resonance images show the characteristic large low-signal masses.

 

Slide 6 shows the typical histologic features of pigmented villonodular synovitis. Note that the macrophages have ingested hemosiderin pigment, giving them a characteristic granular appearance.

The treatment for patients with patient has pigmented villonodular synovitis is synovectomy. For this patient, a reasonable method may be anterior arthroscopic synovectomy and open posterior synovectomy or open anterior and posterior synovectomy.

Correct Answer: Reactive condition

 

 

 

slide 1 slide 2

A 68-year-old woman presents with severe knee pain with ambulation for a 4-month period. Her history is significant for resection and postoperative external beam irradiation for a synovial sarcoma 15 years previously. The anteroposterior and lateral radiographs are shown in Slide 1. The coronal T1- and T2-weighted magnetic resonance images (MRIs) are shown in Slide 2. The most likely diagnosis is:

 

1) Metastatic synovial sarcoma

3) Osteomyelitis

2) Postirradiation sarcoma

5) Osteonecrosis

4) Stress fracture

 

A number of late effects may occur following external beam irradiation for soft tissue sarcomas: postirradiation sarcomas, stress fractures, vascular insufficiency, chronic wound problems, and osteonecrosis.

This patient has a stress fracture of the proximal medial tibia. One can see the lucent area and the condensation of bone in a linear fashion.

The MRI shows very characteristic features:

 

T1-weighted coronal â low signal linear fracture line traversing the tibia

 

T2-weighted coronal â high signal crossing the medial tibia with a low signal linear area (condensation of bone seen in stress fractures)

Nonoperative treatment would be the initial management with protected weight bearing. These stress fractures may require a long time to heal and the nonunion rate is high with nonoperative and operative treatment. If the fracture does not heal in a 3- to 6-month period with nonoperative measures, then rigid internal fixation with or without bone grafting should be considered.

 

Correct Answer: Stress fracture

 

 

 

slide 1 slide 2

A 68-year-old woman presents with severe knee pain with ambulation for a 4-month period. Her history is significant for resection and postoperative external beam irradiation for a synovial sarcoma 15 years previously. The anteroposterior and lateral radiographs are shown in Slide 1. The coronal T1- and T2-weighted magnetic resonance images (MRIs) are shown in Slide 2. What is the most likely etiology of her pain:

 

1) Metastatic disease

3) Bone insufficiency

2) Vascular insufficiency

5) Osteonecrosis

4) Osteoarthritis

 

A number of late effects may occur following external beam irradiation for soft tissue sarcomas: postirradiation sarcomas, stress fractures, vascular insufficiency, chronic wound problems, and osteonecrosis.

This patient has a stress fracture of the proximal medial tibia. One can see the lucent area and the condensation of bone in a linear fashion.

The MRI shows very characteristic features:

 

T1-weighted coronal â low signal linear fracture line traversing the tibia

 

T2-weighted coronal â high signal crossing the medial tibia with a low signal linear area (condensation of bone seen in stress fractures)

Nonoperative treatment would be the initial management with protected weight bearing. These stress fractures may require a long time to heal and the nonunion rate is high with nonoperative and operative treatment. If the fracture does not heal in a 3- to 6-month period with nonoperative measures, then rigid internal fixation with or without bone grafting should be considered.

 

Correct Answer: Bone insufficiency

 

 

3905. (3441) Q9-4366:

Which of the following is the most likely cytotoxic effect of external beam irradiation when treating malignancies:

 

1) Damage to microvasculature

3) Depletion of cellular nutrients

2) Damage to large feeder vessels

5) Hyperthermic effect

4) Creation of free radicals

 

External beam irradiation creates free radicals. The free radicals damage the DNA. When the DNA cannot be repaired, cell death occurs.

 

 

The dose of irradiation is measured in grays or centigrays. A gray is a measure of the dose absorbed â 1 Gy equals 1 J/kg. One should remember the typical dose for delivered for certain tumors:

Correct Answer: Creation of free radicals

 

Which of the following doses is most commonly used as postoperative adjuvant therapy following wide resection of a soft tissue sarcoma:

 

1) 800 cGy

3) 3000 cGy

2) 2000 cGy

5) 6600 cGy

4) 4500 cGy

 

External beam irradiation creates free radicals. The free radicals damage the DNA. When the DNA cannot be repaired, cell death occurs.

 

 

The dose of irradiation is measured in grays or centigrays. A gray is a measure of the dose absorbed â 1 Gy equals 1 J/kg. One should remember the typical dose for delivered for certain tumors:

Correct Answer: 6600 cGy

 

 

3907. (3444) Q9-4370:

Which of the following describes the method of cell toxicity for most chemotherapy drugs:

 

1) Interferes with microtubule function

3) Interrupts adenosine triphosphate-dependent cell membrane pumps

2) Directly damages DNA

5) Induces programmed cell death

4) Depletes cellular building blocks

 

 

Chemotherapy drugs can be classified into three broad classes: Directly damages DNA

 

 

Alkylating agents, platinum compounds, anthracyclines Epipodophyllotoxins

 

 

 

Depletes cellular building blocks Antifolates, cytidine analogs 5-fluoropyrimidines

 

 

Interferes with microtubule function Vinca alkaloids, taxanes

 

Meyers and Gorlick state that, in general, chemotherapy drugs induce programmed cell death (apoptosis). Correct Answer: Induces programmed cell death

 

3908. (3457) Q9-4388:

External beam irradiation is effective in the management of which of the following bone malignancies:

 

1) Osteosarcoma

3) Malignant fibrous histiocytoma

2) Chondrosarcoma

5) Periosteal osteosarcoma

4) Ewingâs tumor

 

 

External beam irradiation can be effective for several malignancies such as: Ewingâs tumor

 

 

Lymphoma Myeloma

 

 

Metastatic bone disease Soft tissue sarcomas

Correct Answer: Ewingâs tumor

 

Which of the following genetic abnormalities occur in fibrodysplasia ossificans progressiva (FOP):

 

1) CBAF1/RUNX2

3) Missense mutation ACVR1

2) Activating mutation of FGFR3

5) Collagen type X mutation

4) Activating missense mutation GNAS (alpha subunit)

 

 

Fibrodysplasia ossificans progressiva is a rare heritable disorder with the following key features: Recurrent episodes of soft tissue swelling that leads to heterotopic ossification

 

 

 

Great toe malformations Incidence of 1 in 2,000,000 Autosomal dominant

 

Genetic findings:

 

Mutation â activin A type 1 receptor gene (ACVR1)

 

Missense mutation in glycine-serine (GS) activation domain The other conditions refer to:

 

 

CBAF1/RUNX2 â cleidocranial dysplasia Activating mutation of FGFR3 â achondroplasia

 

 

Activating missense mutation GNAS (alpha subunit) â fibrous dysplasia Collagen type X mutation â Schmidâs type metaphyseal chondrodysplasia

Correct Answer: Missense mutation ACVR1

 

 

3910. (3494) Q9-4436:

Which of the following is the major regulator of the transition from the G1 phase of the cell cycle to the S phase (synthesis and reproduction):

 

1) p53 protein

3) Cyclin-dependent kinase inhibitors

2) p21 protein

5) Retinoblastoma protein

4) Dihydrofolate reductase (DHFR)

 

During the reproductive cycle of cells, the most important transition is from G1 or G0 (resting states) to the S phase. In the S phase, synthesis of DNA and mitosis occurs. In humans, this transition point is regulated by the retinoblastoma protein.

The retinoblastoma protein is controlled by a group of regulatory proteins â cyclins (cyclin D and E kinases), cyclin-dependent kinases (CDK4, CDK6), and cyclin kinase inhibitors (p16, p21).

To begin cell synthesis, the retinoblastoma protein is phosphorylated by cyclin D and CDK4. This phosphorylation causes release of the E2F transcription factor. The E2F transcription factor signals increased transcription of a number of genes, including dihydrofolate reductase (DHFR), thymidylate synthetase (TS), thymidine kinase (TK), and ribonucleotide reductase (RR). These factors are necessary for DNA replication in the S phase.

 

The important point to remember is that the retinoblastoma gene (Rb1) and the retinoblastoma protein regulate the transition point from G1 to the S phase. Tumor cells cannot grow without going past this transition point.

Correct Answer: Retinoblastoma protein

 

Which of the following statements is true concerning total knee replacement in patients with pigmented villonodular synovitis (PVNS) of the knee:

 

1) Aseptic loosening secondary to recurrent disease is common.

3) Postoperative stiffness is common.

2) Recurrence rates are higher than in patients treated with synovectomy alone.

5) Recurrence rates are low.

4) Postoperative results show failures in more than 50% of patients.

 

Total knee replacement is successful in patients who have PVNS. The results are equal to total knee arthroplasty in patients in the general population and patients who have focal PVNS. Loosening and recurrence are potential complications in a patient who has active diffuse PVNS and undergoes total knee replacement.

 

Important points to remember:

 

 

Interval to arthroplasty, average 8 years (5 months to 23 years) Nine patients had 19 prior procedures

 

Six patients had 14 prior recurrences

 

 

Results (11 patients with active, diffuse disease, average age 59 years): 7/11 intact functioning knees at 11 years

 

2/7 patients required manipulation

 

3/11 required revision for aseptic loosening

 

 

Loosening was not secondary to recurrent PVNS 2/11 recurrence

 

 

One patient required above-knee amputation Correct Answer: Postoperative stiffness is common.

 

3912. (3724) Q9-7504:

Which of the following fusion proteins has the best prognosis in patients with Ewingâs tumor:

 

1) EWS-ERG

3) EWS-ETV1

2) EWS-FLI1

5) EWS-FEV

4) EWS-ETV4

 

Ewingâs tumor is characterized by a balanced translocation â t(11;22). A chimeric protein is formed called the fusion product. The most common fusion product is EWS-FLI1 (found in 85%). EWA-FLI1 has the best prognosis.

The other fusion products found in Ewingâs tumor include:

 

 

 

 

EWS-ERG EWS-ETV1 EWS-ETV4 EWS-FEV

 

Correct Answer: EWS-FLI1

 

Which of the following gene fusion product occurs in Ewingâs tumor:

 

1) SYT-SSX1

3) EWS-FLI1

2) SYT-SSX2

5) EWS-ATF1

4) TLS(FUS)-CHOP

 

Ewingâs tumor is characterized by a balanced translocation â t(11;22). A chimeric protein is formed called the fusion product. The most common fusion product is EWS-FLI1 (found in 85%). EWA-FLI1 has the best prognosis.

The other fusion products found in Ewingâs tumor include:

 

 

 

 

EWS-ERG EWS-ETV1 EWS-ETV4 EWS-FEV

 

The other responses address:

 

Synovial sarcoma

 

  • SYT-SSX1

  • SYT-SSX2

 

 

Myxoid liposarcoma â TLS (FUS)-CHOP Clear cell sarcoma

Correct Answer: EWS-FLI1

 

 

3914. (3726) Q9-7506:

Which of the following gene fusion products occurs in patients with synovial sarcoma:

 

1) EWS-FLI1

3) EWS-ATF1

2) TLS (FUS) â CHOP

5) EWS-ERG

4) SYT-SSX1

 

Synovial sarcoma has a characteristic translocation â t(X;18). The gene fusion product is SYT-SSX1 and SYT-SSX2. The other responses address:

 

Ewings sarcoma

 

     o  EWS-FLI1      o  EWS-ERG

 

 

Myxoid liposarcoma â TLS (FUS)-CHOP Clear cell sarcoma â EWS-ATF1

Correct Answer: SYT-SSX1

 

Which of the following gene fusion products occurs in patients with clear cell sarcoma:

 

1) EWS-FLI1

3) EWS-ATF1

2) TLS (FUS)-CHOP

5) SYT-SSX2

4) SYT-SSX1

 

Clear cell sarcoma has a balanced translocation â t(12;22). The gene fusion product is TLS (FUS)-CHOP. The other responses address:

 

Synovial sarcoma

 

     o   SYT-SSX1      o  SYT-SSX2

 

 

Ewings tumor â EWS-FLI1 Correct Answer: TLS (FUS)-CHOP

 

3916. (3743) Q9-7523:

Which of the following is the best approximate survival of patients with breast cancer following metastasis to bone:

 

1) 6 months

3) 24-36 months

2) 12 months

5) 10 years

4) 5 years

 

The average survival of patients with breast cancer is between 24-36 months. The approximate survival for patients with the other common tumors is:

 

 

Lung, kidney, melanoma: 6 months Prostate: 48 months

Correct Answer: 24-36 months

 

 

3917. (3744) Q9-7524:

Which of the following is released by metastatic breast cancer cells in bone and activates osteoclast precursor cells:

 

1) Receptor activator of nuclear factor kappa B ligand (RANKL)

3) Metalloproteinases (MMPs)

2) Osteoprotegerin (OPG)

5) Parathyroid hormone-related protein (PTHrP)

4) Tumor necrosis factor (TNF)

 

The metastatic breast cancer cells release parathyroid hormone-related protein (PTHrP), which stimulates osteoblasts to release the receptor activator of nuclear factor kappa B ligand (RANKL). RANKL attaches to the receptor activator of nuclear factor kappa B (RANK) receptor on the osteoclast precursor cells and initiates osteoclast activation.

 

The other responses refer to:

 

MMPs â Released by tumor cells to cause dissolution of the basement membrane and matrices

 

TNF â Released in rheumatoid arthritis, resulting in synovial proliferation and articular cartilage damage

 

OPG â A competitive decoy inhibitor that binds to the RANKL and prevents attachment to the RANK receptor on the osteoclast precursor cells; OPG prevents osteoclast activation and bone resorption

Correct Answer: Parathyroid hormone-related protein (PTHrP)

 

Which of the following is released by breast cancer cells and causes direct activation of osteoclasts:

 

1) Parathyroid hormone-related protein (PTHrP)

3) Interleukin-8 (IL-8)

2) Osteoprotegerin (OPG)

5) MIP-1α

4) Endothelin-1

 

Interleukin-8 (IL-8) is released by metastatic breast cancer cells and can cause direct activation of osteoclasts. This direct activation is independent of the RANKL activation of the RANK receptor.

The other responses refer to:

 

PTHrP â Released by metastatic breast cancer cells and causes activation of the osteoclast via signaling to the osteoblast for release of the RANKL

 

 

 

OPG â A competitive decoy inhibitor that binds to the RANKL and prevents attachment to the RANK receptor on the osteoclast precursor cells; OPG prevents osteoclast activation and bone resorption Endothelin-1 â Causes bone formation in metastatic prostate cancer to bone

 

 

 

MIP-1α â Produced by multiple myeloma cells and causes osteoclast activation Correct Answer: Interleukin-8 (IL-8)

 

3919. (4065) Q9-7526:

Which of the following causes bone formation in metastatic prostate cancer:

 

1) Parathyroid hormone-related protein (PTHrP)

3) Endothelin-1

2) Interleukin-8 (IL-8)

5) Osteoprotegerin (OPG)

4) MIP-1α

 

Endothelin-1 is postulated to be responsible for the osteoblastic metastases in prostate cancer. The other responses refer to:

 

PTHrP â Released by metastatic breast cancer cells and causes activation of the osteoclast via signaling to the osteoblast for release of the RANKL

 

OPG â A competitive decoy inhibitor that binds to the RANKL and prevents attachment to the RANK receptor on the osteoclast precursor cells; OPG prevents osteoclast activation and bone resorption

 

 

MIP-1α â Produced by multiple myeloma cells and causes osteoclast activation IL-8 â Causes direct activation of osteoclasts and bone resorption

Correct Answer: Endothelin-1

 

 

3920. (3746) Q9-7527:

Which of the following is the best approximation of survival after bone metastases in lung cancer:

 

1) 6 months

3) 36 months

2) 24 months

5) 60 months

4) 48 months

 

 

The average survival of patients with lung cancer following bone metastases is approximately 6 months. Correct Answer: 6 months

Which of the following carcinomas has a propensity to metastasize to distal sites such as the hands and feet:

 

1) Breast cancer

3) Lung cancer

2) Kidney cancer

5) Prostate cancer

4) Thyroid cancer

 

 

Lung cancer has a predilection to metastasize to distal sites such as the hands and feet. Correct Answer: Lung cancer

 

3922. (3748) Q9-7529:

Which of the following directly promotes tumor cell metastasis:

 

1) NM-23

3) Receptor activator of nuclear factor kappa B ligand (RANKL)

2) MKK4

5) RKIP

4) Activated RAS

 

Activated RAS begins the Raf-MEK-ERK-MAPK pathway, which causes tumor cell growth and metastasis. Tumor suppressor genes can inhibit the metastatic process:

 

 

 

NM-23 â phosphorylates the Ksr protein RKIP â competitive inhibitor of MEK MKK4 â causes stress-induced apoptosis

 

 

RANKL binds to the RANK receptor and activates the osteoclast precursor cells. Correct Answer: Activated RAS

 

3923. (3749) Q9-7530:

Which of the following directly promotes tumor cell metastasis:

 

1) Receptor activator of nuclear factor kappa B ligand (RANKL)

3) Osteoprotegerin (OPG)

2) Receptor activator of nuclear factor kappa B (RANK)

5) Activated RAS

4) Tumor necrosis factor (TNF)

 

Activated RAS begins the Raf-MEK-ERK-MAPK pathway, which causes tumor cell growth and metastasis.

RANKL binds to the RANK receptor and activates the osteoclast precursor cells. Osteoprotegerin is a competitive inhibitor of the RANKL (binds it and promotes the RANKL attaching to the RANK receptor).

 

Tumor necrosis factor is an inflammatory factor causing destruction of the articular cartilage. Correct Answer: Activated RAS

Which of the following causes metastatic cells to localize to bone (homing in mechanism):

 

1) NM-23

3) RKIP

2) MKK4

5) PTHrP

4) CXCL12

 

CXCL12 is called chemokine ligand 12 (stromal cell-derived factor 1). This factor is thought to be responsible for metastatic tumor cells to localize to bone. Chemostasis occurs as chemokine receptor 4 is activated on the tumor cells.

 

Tumor suppressor genes can inhibit the metastatic process: NM-23 â phosphorylates the Ksr protein

 

 

RKIP â competitive inhibitor of MEK MKK4 â causes stress-induced apoptosis

Parathyroid hormone-related protein (PTHrP) is released by breast cancer metastatic cells. The same receptors for parathyroid hormone (PTH) are activated.

 

PTH/PTHrP causes osteoclast activation and bone resorption.

Correct Answer: CXCL12

 

 

3925. (3751) Q9-7532:

Which of the following is important for cell attachment in the metastatic process:

 

1) Receptor activator of nuclear factor kappa B ligand (RANKL)

3) Integrins

2) Receptor activator of nuclear factor kappa B (RANK)

5) CXCL12

4) Metalloproteinases (MMPs)

 

The integrins are the most common of all the cell adhesion molecules. Adhesion molecules allow tumor cells to attach to the basement membrane for eventual penetration. The integrins also play an important role in tumor cells attaching to bone.

Examples:

 

 

Alpha âv-beta 3 (άυβ3) â breast cancer cells attaching to type I collagen Alpha âv-beta 1 (άυβ1) â osteoclast binding to bone

Correct Answer: Integrins

 

 

3926. (3752) Q9-7533:

Which of the following factors is important for invasion of the basement membrane in the metastatic process:

 

1) Integrins

3) Parathyroid hormone-related protein (PTHrP)

2) Metalloproteinases (MMPs)

5) Receptor activator of nuclear factor kappa B ligand (RANKL)

4) CXCL12

 

Metalloproteinases (MMPs) are a group of enzymes that break down matrixes and membranes. The collagenases are in the group of MMPs. The MMPs are important for penetration of the basement membrane.

The other responses refer to:

 

Integrins â Adhesive proteins for cell attachment

 

 

PTHrP â Causes osteoclast activation and bone resorption CXCL12 â Causes homing of tumor cells to bone

 

 

RANKL â Causes activation of osteoclast precursor cells to osteoclasts Correct Answer: Metalloproteinases (MMPs)

3927. (3753) Q9-7534:

Which of the following factors is important for angiogenesis in the metastatic process:

 

1) Vascular endothelial growth factor (VEGF)

3) CXCL12

2) Parathyroid hormone-related protein (PTHrP)

5) Receptor activator of nuclear factor kappa B ligand (RANKL)

4) Integrins

 

Vascular endothelial growth factor is an important stimulant for new blood vessel formation. The matrix is first broken down by matrix degrading enzymes such as plasminogen activator and metalloproteinases.

The other responses refer to:

 

Integrins â Adhesive protein for cell attachment

 

 

PTHrP â Causes osteoclast activation and bone resorption CXCL12 â Causes homing of tumor cells to bone

 

 

RANKL â Causes activation of osteoclast precursor cells to osteoclasts Correct Answer: Vascular endothelial growth factor (VEGF)

 

3928. (3855) Q9-7638:

Which of the following diagnoses is most consistent with a magnetic resonance image that shows high peripheral signal intensity and low central intensity on T2-weighted images and low signal on T1-weighted images:

 

1) Nodular fasciitis

3) Malignant fibrous histiocytoma

2) Schwannoma

5) Epitheloid sarcoma

4) Synovial sarcoma

 

Schwannomas have characteristic findings on magnetic resonance scans. These benign nerve sheath tumors sometimes have a characteristic signal appearance on T2-weighted images, which includes a high peripheral signal intensity and low central intensity on T2-weighted images.

 

 

This can also be called a target sign. Correct Answer: Schwannoma

 

3929. (3856) Q9-7639:

Which of the following soft tissue tumors may have a split-fat sign on T1-weighted images around the tumor:

 

1) Nodular fasciitis

3) Synovial sarcoma

2) Malignant fibrous histiocytoma

5) Schwannoma

4) Epitheloid sarcoma

 

The split-fat sign on T1-weighted images occurs when a thin rim of fat is present around a mass. Although this finding is nonspecific, it commonly occurs in schwannomas.

Correct Answer: Schwannoma